mental health study guide

Réussis tes devoirs et examens dès maintenant avec Quizwiz!

Which behaviors observed by the nurse might lead to the suspicion that a depressed adolescent client could be suicidal?

The client gives away a DVD and a cherished autographed picture of the performer.

This class of medications is used PRN for anxiety, but also for ETOH withdrawal, seizures, and as an adjunct for surgery/procedure

benzodiazepines

main concerns c carbamazepine:

blood dyscrasias - infections - bleeds double vision, vertigo fluid overload NOT for pregnancy (also decreases OCP effectiveness)

which antidepressant also has a formulation used for smoking cessation

bupropion (Zyban is for smoking cessation, Wellbutrin is for depression)

Inform the client that the provider can prescribe an atypical antidepressant instead of SSRIs with fewer sexual dysfunction adverse effects, such as _____________.

bupropion.

lithium: give c food or on empty stomach?

c food - can cause GI upset usually subsides after taking it for a while

3 anti-seizure meds used for mood stabilizing?

carbamazepine (Tegretol) valproate (Depakote) lamotrigine (Lamictal)

Quetiapine can cause...

cataracts

ETOH withdrawal: _________ used for acute withdrawal, __________ used for long-term maintenance

chlordiazepoxide (Librium) disulfiram (Antabuse)

medication given for neuroleptic malignant syndrome?

dantrolene (muscle relaxer) (also used for malignant hyperthermia)

benzos that can be administered IV

diazepam (Valium) lorazepam (Ativan)

pt tells you they're taking St John's Wort - what is a concern?

drug interactions - serotonin syndrome risk c SSRIs, MAOIs, tramadol, Talwin - dec effectiveness of OCPs - dec effectiveness of warfarin and digoxin - dec effectiveness of phenytoin (more seizures)

Anticholinergic effects of Tricyclic antidepressants

dry mouth blurred vision photophobia urinary hesitancy or retention constipation tachycardia

Indicator of TCA toxicity?

dysrhythmias

During an individual therapy session, a nurse is listening to a client describe the client's drug addiction. The client says, "I know I am doing the wrong thing for my kids, but I just can't stop using drugs." The nurse maintains eye contact and nods occasionally. The nurse responds by saying, "You're going through a difficult time." The nurse's actions and words are an example of:

empathy

ADHD meds: CNS stimulants

methylphenidate amphetamine salts

Main AE to watch for c lamotrigine

rash - can be benign or can be SJS - usually d/c for any rash

The nurse is assessing a newly admitted client recently diagnosed with depression. Which data best supports that the client is at risk for self-harm?

reported hopelessness

Laboratory work is prescribed for a client who has been experiencing delusions. When the laboratory technician approaches the client to obtain a specimen of the client's blood, the client begins to shout, "You're all vampires. Let me out of here!" The nurse present at the time should respond with which question or statement?

"Are you fearful and think that others may want to hurt you?"

When a client states, "I will solve my own problems without asking my family for help," which response by the nurse demonstrates a therapeutic use of self?

"Asking for help from those who care about us isn't a sign of weakness."

An unlicensed assistive personnel (UAP) is assigned to work with the nurse to care for a client who is at risk for suicide. Which statement made by the UAP indicates to the nurse that the UAP understands suicide?

"Discussing suicide with a client is not harmful."

The nurse is preparing for the hospital discharge of a client with a history of command hallucinations to harm self or others. The nurse instructs the client about interventions for hallucinations and anxiety and determines that the client understands the interventions when the client makes which statement?

"I can call my therapist when I'm hallucinating so I can talk about my feelings and plans and not hurt anyone."

The nurse is monitoring a client who is in seclusion. Which statement would indicate that the client is safe to come out of seclusion?

"I don't feel like hurting myself anymore."

A long-term care resident with a history of paranoid schizophrenia refuses to eat and tells the nurse that she believes that someone is poisoning the food. The nurse should make which therapeutic response to the client?

"It must be frightening to you. Has something made you feel that your food is poisoned?"

A client states to the nurse, "I haven't slept at all the last couple of nights." The nurse should make which therapeutic response to the client?

"Tell me about your difficulty sleeping."

During the next meeting during the working phase of the relationship the client brings the nurse homemade chocolate chip cookies and a box of chocolates. Which response should the nurse make to the client about these gifts?

"Thank you but I will not accept these gifts because they extend over our discussed boundaries."

Which statement is the most empathic response to a client's disclosure that the client's father abandoned the family when the client was a young child?

"That must have been terribly hurtful experience for you."

The nurse is caring for an older adult client who has recently lost her husband. The client says, "No one cares about me anymore. All the people I loved are dead." Which response by the nurse is therapeutic?

"You must be feeling all alone at this point."

A client who has just received a diagnosis of asthma says to the nurse, "This condition is just another nail in my coffin." Which response by the nurse is therapeutic?

"You seem very distressed over learning you have asthma."

A client says to the nurse, "I'm going to die, and I wish my family would stop hoping for a 'cure'! I get so angry when they carry on like this! After all, I'm the one who's dying." Which therapeutic response should the nurse make to the client?

"You're feeling angry that your family continues to hope for you to be 'cured'?"

A client with depression who has attempted suicide says to the nurse, "I should have died. I've always been a failure. Nothing ever goes right for me." The nurse should make which therapeutic response to the client?

"You've been feeling like a failure for a while?"

A nurse is speaking to a client with a soft smile and eye contact. Which statement said by the nurse would indicate a congruent message?

"Your hard work and determination has helped you recover."

Complications with SSRIs for depression disorders

*Sexual Dysfunction* *CNS stimulation*: insomnia, agitation, anxiety Weight changes (initial loss then gain) *Serotonin syndrome* Withdrawal syndrome *Hyponatremia* esp in older adults on diuretics Rash Sleepiness, faintness, lightheadedness GI bleeding *Bruxism* esp c citalopram

what are EPSs?

*akathisia* - restlessness, pacing *dystonia* - muscle spasms *parkinsonism* - bradykinesia, rigidity, tremor

main lithium drug interactions

- diuretics d/t Na loss - NSAIDs (exception: ASA is a-okay!) - anticholinergics (starts an uncomfortable internal war over polyuria vs. urine retention)

Main 2nd gen AP concerns

- metabolic syndrome - weight, lipids, blood sugar - ACh effects (including orthostatic hypotension) - Sexual dysfunction - Prolactin levels - Mild EPS eg tremor

don't mix benzos with...

- other CNS depressants like ETOH (can kill you) - CNS stimulants like caffeine (kinda cancel each other out)

how to d/c SSRIs

- slowly - taper off - withdrawal s/s - nausea, anxiety, tremor, unease - paroxetine is notoriously hard to d/c

Serotonin syndrome and NMS sound a lot alike. What's the difference?

- type of medication causing it - onset: SS within a day, NMS a few days - SS: *muscles contracting, hyperreflexia* - NMS: *muscles rigid, hyporeflexia*

A client with a history of victim abuse has which signs/symptoms of the physical effects of living with a severe level of anxiety and chronic stress? Select all that apply.

-Irritability -Hypertension -GI disturbances

When conducting a psycho-social assessment, the nurse inquires about the client's social supports. In order to effectively do this, which does the nurse need to explore?

. The length and quality of relationships

The nurse is planning care for a client being admitted to the nursing unit who attempted suicide. Which priority nursing intervention should the nurse include in the plan of care? 1. One-to-one suicide precautions 2. Suicide precautions with 30-minute checks 3. Checking the whereabouts of the client every 15 minutes 4. Asking the client to report suicidal thoughts immediately

1. One-to-one suicide precautions Rationale: One-to-one suicide precautions are required for a client who has attempted suicide. Options 2 and 3 may be appropriate, but not at the present time, considering the situation. Option 4 also may be an appropriate nursing intervention, but the priority is identified in the correct option. The best intervention is constant supervision so that the nurse may intervene as needed if the client attempts to harm himself or herself.

What should the nurse avoid when demonstrating genuine interest for a client by making a self-disclosure

shifting the emphasis to the nurse

TCAs more likely to be prescribed for ----- than depression anymore?

sleep problems, chronic pain

The spouse of a client admitted to the mental health unit for alcohol withdrawal says to the nurse, "I should get out of this bad situation." What is the most helpful response by the nurse? 1. "Why don't you tell your wife about this?" 2. "What do you find difficult about this situation?" 3. "This is not the best time to make that decision." 4. "I agree with you. You should get out of this situation."

2. "What do you find difficult about this situation?" Rationale: The most helpful response is one that encourages the client to solve problems. Giving advice implies that the nurse knows what is best and can foster dependency. The nurse should not agree with the client, and the nurse should not request that the client provide explanations.

The nurse in the mental health unit is assigned to care for a female client with a diagnosis of acute depression. In communicating with the client, which statement would be appropriate for the nurse to make? 1. "You look lovely today." 2. "You're wearing a new blouse." 3. "Don't worry-everyone gets depressed once in a while." 4. "You will feel better when your medication starts to work."

2. "You're wearing a new blouse." Rationale: A client who is depressed sees the negative side of everything. Telling the client that she looks lovely today can be interpreted as "didn't look lovely last time we met." Neutral comments such as that identified in the correct option will avoid negative interpretations. The client should not be told not to worry, that everyone gets depressed once in a while, or that he or she will feel better, because such statements are inappropriate.

A client with a diagnosis of anorexia nervosa, who is in a state of starvation, is in a two-bed room. A newly admitted client will be assigned to this client's room. Which client would be the best choice as a roommate for the client with anorexia nervosa? 1. A client with pneumonia 2. A client undergoing diagnostic tests 3. A client who thrives on managing others 4. A client who could benefit from the client's assistance at mealtime

2. A client undergoing diagnostic tests Rationale: The client undergoing diagnostic tests is an acceptable roommate. The client with anorexia nervosa is most likely experiencing hematological complications, such as leukopenia. Having a roommate with pneumonia would place the client with anorexia nervosa at risk for infection. The client with anorexia nervosa should not be put in a situation in which the client can focus on the nutritional needs of others or be managed by others because this may contribute to sublimation and suppression of personal hunger.

The nurse is caring for a client with a diagnosis of agoraphobia. When communicating with the client about the disorder, the nurse should expect the client to describe which behavior? 1. A fear of dirt and germs 2. A fear of leaving the house 3. A fear of speaking in public 4. A fear of riding in elevators

2. A fear of leaving the house Rationale: Agoraphobia is a fear of leaving the house and experiencing panic attacks when doing so. Option 1 describes an obsessive-compulsive behavior. Option 3 describes a social phobia. Option 4 describes claustrophobia.

The nurse is caring for a client who is experiencing disturbed thought processes as a result of paranoia. In formulating nursing interventions with the members of the health care team, what best instruction should the nurse provide to the staff? 1. Increase socialization of the client with peers. 2. Avoid laughing or whispering in front of the client. 3. Begin to educate the client about social supports in the community. 4. Have the client sign a release of information to appropriate parties for assessment purposes.

2. Avoid laughing or whispering in front of the client. Rationale: Disturbed thought process related to paranoia is the client's problem, and the plan of care must address this problem. The client is experiencing paranoia and is distrustful and suspicious of others. The members of the health care team need to establish a rapport and trust with the client. Laughing or whispering in front of the client would be counterproductive. The remaining options ask the client to trust on a multitude of levels. These options are actions that are too intrusive for a client who is paranoid.

The health care provider has prescribed medication therapy for a client with an alcohol abuse problem to assist in the maintenance of sobriety. The nurse reviews the client's record and expects to note that which medication has been prescribed? 1. Clonidine (Catapres) 2. Disulfiram (Antabuse) 3. Pyridoxine hydrochloride (vitamin B6) 4. Chlordiazepoxide hydrochloride (Librium)

2. Disulfiram (Antabuse) Rationale: Disulfiram is a medication used for alcoholism, and it aids in the maintenance of sobriety. Clonidine is an antihypertensive medication. Pyridoxine hydrochloride is used in the treatment of vitamin B6 deficiency. Chlordiazepoxide hydrochloride is an antianxiety medication (a benzodiazepine) that is used in the management of acute alcohol withdrawal symptoms.

A client with anorexia nervosa is a member of a predischarge support group. The client verbalizes that she would like to buy some new clothes, but her finances are limited. Group members have brought some used clothes to the client to replace the client's old clothes. The client believes that the new clothes were much too tight and has reduced her calorie intake to 800 calories daily. How should the nurse evaluate this behavior? 1. Normal behavior 2. Evidence of the client's disturbed body image 3. Regression as the client is moving toward the community 4. Indicative of the client's ambivalence about hospital discharge

2. Evidence of the client's disturbed body image Rationale: Disturbed body image is a concern with clients with anorexia nervosa. Although the client may struggle with ambivalence and show regressed behavior, the client's coping pattern relates to the basic issue of disturbed body image. The nurse should address this need in the support group.

The police arrive at the emergency department with a client who has lacerated both wrists. What is the initial nursing action? 1. Administer an antianxiety agent. 2. Examine and treat the wound sites. 3. Secure and record a detailed history. 4. Encourage and assist the client to ventilate feelings.

2. Examine and treat the wound sites. Rationale: The initial nursing action is to assess and treat the self-inflicted injuries. Injuries from lacerated wrists can lead to a life-threatening situation. Other interventions, such as options 1, 3, and 4, may follow after the client has been treated medically.

When planning the discharge of a client with chronic anxiety, the nurse directs the goals at promoting a safe environment at home. Which is the most appropriate maintenance goal? 1. Suppressing feelings of anxiety 2. Identifying anxiety-producing situations 3. Continued contact with a crisis counselor 4. Eliminating all anxiety from daily situations

2. Identifying anxiety-producing situations Rationale: Recognizing situations that produce anxiety allows the client to prepare to cope with anxiety or avoid a specific stimulus. Counselors will not be available for all anxiety-producing situations, and this option does not encourage the development of internal strengths. Suppressing feelings will not resolve anxiety. Elimination of all anxiety from life is impossible.

The nurse is preparing a client for the termination phase of the nurse-client relationship. The nurse prepares to implement which nursing task that is most appropriate for this phase? 1. Planning short-term goals 2. Making appointment referrals 3. Developing realistic solutions 4. Identifying expected outcomes

2. Making appointment referrals Rationale: Tasks of the termination phase include evaluating client performance, evaluating achievement of expected outcomes, evaluating future needs, making appropriate referrals, and dealing with the common behaviors associated with termination. The remaining options identify tasks appropriate for the working phase of the relationship.

The nurse is caring for a client with anorexia nervosa. Which behavior is characteristic of this disorder and reflects anxiety management? 1. Engaging in immoral acts 2. Always reinforcing self-approval 3. Observing rigid rules and regulations 4. Having the need always to make the right decision

3. Observing rigid rules and regulations Rationale: Clients with anorexia nervosa have the desire to please others. Their need to be correct or perfect interferes with rational decision-making processes. These clients are moralistic. Rules and rituals help these clients manage their anxiety.

A depressed client on an inpatient unit says to the nurse, "My family would be better off without me." What is the nurse's best response? 1. "Have you talked to your family about this?" 2. "Everyone feels this way when they are depressed." 3. "You will feel better once your medication begins to work." 4. "You sound very upset. Are you thinking of hurting yourself?"

4. "You sound very upset. Are you thinking of hurting yourself?" Rationale: Clients who are depressed may be at risk for suicide. It is critical for the nurse to assess suicidal ideation and plan. The nurse should ask the client directly whether a plan for self-harm exists. Options 1, 2, and 3 do not deal directly with the client's feelings.

The nurse employed in a psychiatric unit receives a client assignment for the day. Which client assigned to the nurse is at the highest risk for committing suicide?

A client with severe depression and terminal cancer

A confused and disoriented client is admitted to the psychiatric unit diagnosed with posttraumatic stress disorder (PTSD). The nurse initially plans to take which action with this client?

Accept the client as a person and make the client feel safe.

Which role of the nurse-client relationship is being exhibited when the nurse informs the client and then supports the client in whatever decision the client makes?

Advocate

A client is unwilling to get out of the house for fear of "doing something crazy in public." Because of this fear, the client remains homebound except when accompanied outside by the spouse. The spouse asks the nurse, "what is the name of my wife's disorder?" Which answer should the nurse give to the spouse?

Agoraphobia

A client who has been drinking alcohol on a regular basis admits to having "a problem" and is asking for assistance with the problem. The nurse should encourage the client to attend which community group?

Alcoholics Anonymous

A client is scheduled to have electroconvulsive therapy (ECT). Which information should the nurse tell the client?

Amnesia of events occurring near the period of the therapy is common.

A client is admitted to the psychiatric unit after a serious suicidal attempt by hanging. What is the nurse's most important intervention to maintain client safety?

Assign a staff member to the client who will remain with him or her at all times.

The nurse is planning care for a client who is being hospitalized because the client has been displaying violent behavior and is at risk for potential harm to others. The nurse should avoid which intervention in the plan of care?

Assigning the client to a room at the end of the hall to prevent disturbing the other clients

The nurse is preparing a client for the termination phase of the nurse-client relationship. Which task should the nurse appropriately plan for during this phase?

Assist with making appropraite referrals

A licensed practical nurse (LPN) is caring for a client with a diagnosis of schizophrenia. The LPN observes behaviors indicative of paranoia and reports these observations to the registered nurse (RN). The LPN assists the RN in developing a plan of care for the client and suggests inclusion of which intervention in the plan of care?

Avoid joking or laughing in the presence of the client.

A hospitalized client with a history of alcohol abuse tells the nurse, "I am leaving now. I have to go. I don't want any more treatment. I have things that I have to do right away." The client has not been discharged. In fact, the client is scheduled for an important diagnostic test to be performed in 1 hour. After the nurse discusses the client's concerns with the client, the client dresses and begins to walk out of the hospital room. Which is the appropriate nursing action?

Call the nursing supervisor

A 68-year-old parent is the sole care provider for a 39-year-old child who has a diagnosis of bipolar disorder. The 39-year-old has been experiencing worsening of the illness over several years. The nurse should recognize that the parent is at risk for what?

Compassion fatigue

The nurse has a client who seems like the nurse's sister, with whom the nurse has a close and positive relationship. This phenomenon is best characterized by which term?

Countertransference

The nurse is assisting in developing a plan of care for a paranoid client who experiences religious delusions. Which short-term goal would be most appropriate?

Develops a relationship to help reduce the frequency of the delusions

The nurse is caring for a client with severe depression. Which activity is appropriate for this client?

Drawing

In what phase of the therapeutic relationship does the assessment process begin?

During the initiating or orienting phase

The nurse is assisting in conducting a group therapy session. A client who has shared with the group at a previous session that she isolates herself when she feels depressed, suddenly gets up to leave. Which nursing action is appropriate?

Encourage the client to stay and ask the client what she is feeling.

The nurse is assigned to assist in the care of a client with obsessive-compulsive disorder (OCD). The nurse should place priority on which action when planning care for this client?

Establish a trusting nurse-client relationship.

The police arrive at the emergency department with a client who has seriously lacerated both wrists. Which is the initial nursing action?

Examine and treat the wound sites.

A 20-year-old client who has a diagnosis of schizophrenia frequently experiences delusions of persecution. At the prompting of the client's mother, the nurse attempts to determine the character and severity of these delusions on a particular day. In doing so, the nurse is conducting what type of assessment?

Focused

APs have risk for neuroleptic malignant syndrome. S/S?

HTN, hyperthermia, tachycardia, tachypnea Altered LOC possible Hyporeflexia Muscle rigidity

The nurse is assisting in admitting a client with schizophrenia to an acute-care inpatient psychiatric unit from the emergency department; however, the client refuses admission. Which intervention should the nurse implement?

Help the client with problem solving.

Which theorist was most widely known for the belief that the cornerstone of all nursing care is the therapeutic relationship?

Hildegard Peplau

An adolescent who has been reported for drawing sexually explicit scenes in her school textbooks says to the psychiatric nurse, "I just felt like it." Which response is therapeutic for the nurse to make in order to assess abuse-related symptoms?

I am concerned about you. Are you...

A manic client is placed in a seclusion room after an outburst of violent behavior, including physical assault on another client. As the client is secluded, which action should the nurse perform?

Inform the client that she is being secluded to help regain control of herself.

The nurse employed in an emergency department is assisting in caring for an adult client who is a victim of family violence. The nurse reinforces which instruction to the victim in the discharge plan?

Information regarding the location of shelters

The nurse has been closely observing a client who has been displaying aggressive behaviors. The nurse observes that the behavior displayed by the client is escalating. Which nursing intervention is least likely to be helpful to this client at this time?

Initiate confinement measures.

atomoxetine onset?

takes 1-3 weeks

lithium onset?

takes 5-7 days! can also give a faster-onset antipsychotic like Zyprexa to bridge until therapeutic Li levels are reached

A nurse interviews a new client in the day room of the psychiatric unit. The nurse is wearing a jacket and a bag and frequently asks the client to repeat the last statement. The nurse's demeanor with the client is reflective of what?

Lacking genuine interest

1st gen APs are classified as low, medium, or high potency. This means?

Low: *low* EPS, *high* sedation, *high* ACh Med: *med* EPS, *med* sedation, *low* ACh High: *high* EPS, *low* sedation, *low* ACh

When taking ________ avoid taking OTC decongestants and cold remedies.

MAOIs

The nursing student is creating a plan of care for the hospitalized client with bulimia nervosa. The nursing instructor intervenes if the student documents which intervention in the plan that is not specific to this disorder?

Observe for excessive exercise.

The nurse is monitoring the behavior of the client and understands that the client with anorexia nervosa manages anxiety by which action?

Observing rigid rules and regulations

It is the nurse's responsibility to define the boundaries of the relationship during which phase of the nurse-client relationship?

Orientation

The psychiatric nurse correctly identifies the client's form of communication as circumstantiality when the client does what?

Provides long, irrelevant explanations when asked why the client abuses alcohol.

The nurse reviews the treatment prescribed for a client with a mental health disorder. The nurse understands that a form of psychotherapy in which the client enacts situations that are of emotional significance is identified by which term?

Psychodrama

Which is not a goal of the working phase of the therapeutic relationship?

Reducing the client's anxieties

A client is admitted to the psychiatric unit following a serious suicide attempt by a drug overdose. Which action should the nurse implement?

Remain with the client at all times

A nurse is conducting a 6-week social skills training program. A young adult with schizophrenia asks the nurse to call the client on the weekends so the client has someone to talk to who really cares. Which action should the nurse take?

Remind the client about the importance of boundaries to keep the relationship therapeutic

A client with obsessive-compulsive disorder (OCD) who continually cleans the bathroom becomes enraged with the roommate for using the bar of bathing soap for cleaning the bathroom. The client begins to yell and slaps the roommate. Which action should the nurse take first?

Remove both clients to a separate, safe location.

The nurse is caring for a client who has been identified as a victim of physical abuse. In planning care for the client, which nursing action is the priority?

Removing the client from any immediate danger

A client is scheduled to have electroconvulsive therapy (ECT). Which problem should the nurse include in the plan as a priority?

Risk for aspiration

Ziprasidone can be used in pts who have both...

SCZ and depression

Warn the client of possible sexual dysfunction and to notify the provider if they become intolerable when taking ________ for depression disorders.

SSRIs

Weight loss early in therapy that can be followed by weight gain with long term treatment when taking ________.

SSRIs

first line drug tx for OCD?

SSRIs - paroxetine, fluvoxamine - at high doses

A client experiencing delusions of being poisoned is admitted to the hospital after not eating or drinking for several days. On data collection, the nurse notes no evidence of dehydration and malnutrition at this time. The nurse should immediately plan to address which client need?

Safety and Security

The nurse notices a "paranoid stare" during a conversation with the client diagnosed with posttraumatic stress disorder (PTSD). The client then begins to fidget and gets up to pace around the room. Which action by the nurse would be most beneficial?

Share the observation with the client and help the client recognize his or her feelings.

The nurse is collecting data from a client recently diagnosed with paranoid schizophrenia. Which information best supports that the client is at risk for harming another individual?

Sibling stating, "I don't feel safe around my brother."

The nurse is looking to assess the client's ability to concentrate. Which task should the nurse ask the client to perform?

Spell "America" backward

Which nursing approach is important when administering an antianxiety agent to a client with acute, severe anxiety?

Stay with the client until the medication becomes effective.

A female victim of a sexual assault is being seen in the crisis center. The client states that she still feels "as though the rape just happened yesterday," even though it has been a few months since the incident. Which nursing response is appropriate?

Tell me more about what causes you to...

During the working phase, a client demonstrates open hostility in reaction to the nurse's last question. Which response should the nurse make to avoid countertransference? "

Tell me why you are angry about what I just said."

A nurse is engaged in a therapeutic nurse-client relationship. The relationship is in the working phase. With which would the client be involved?

Testing new ways for problem solving Discussing problems related to needs Examining personal issues

When engaged in a therapeutic relationship, the nurse's focus is on what?

The Client

The nurse is monitoring a client with a diagnosis of depression. Which behavior observed by the nurse indicates that suicide precautions should be instituted for this client?

The client asks to meet with a lawyer to take care of unfinished business.

The nurse is performing an assessment of a client with psychiatric illness. The nurse documents that the client has a restricted affect. Which behavior of the client is indicative of restricted affect?

The client displays only one type of facial expression.

Which statement would indicate that the nurse has a non-judgmental attitude?

The client has struggled with her life circumstance of living with a man who beats her, and she is trying very hard to make the changes necessary to help herself."

The nurse is reviewing the record of a client admitted to the mental health unit and notes that the client was admitted by voluntary status. The nurse makes which determination?

The client has the right to demand ...

A nurse is meeting a client for the first time. The nurse observes that the client smiles appropriately but is using rambling speech while answering the nurse's questions. Which would most likely be the reason for this behavior?

The client is nervous and insecure.

Which would indicate that the nurse-client relationship has passed from the orienting phase to the working phase?

The client recognizes feelings of anger and expresses them appropriately.

A hospitalized client who recently experienced the loss of a spouse is grieving. The client progresses well and is approaching discharge. Which is an appropriate outcome for this client?

The client verbalizes stages of grief and plans to attend a community grief group.

The nurse in a psychiatric unit is assigned to care for a client admitted to the unit 2 days ago. During review of the client's record, the nurse notes that the admission was a voluntary one. Based on this type of admission, which would the nurse expect to note?

The client will participate in the treatment plan.

The nurse notes documentation in a client's record that the client is experiencing delusions of persecution. The nurse recognizes that these types of delusions are characteristic of which thoughts?

The false belief that one is being singled out for harm by others

The nurse finds that the client is constantly rubbing the hands. Under which component of psychosocial assessment should the nurse document this finding?

The general assessment and motor behavior component

A client reveals in a therapy session that the client has thought about killing a neighbor. What is the therapist's obligation regarding this revelation?

The therapist must notify authorities and the potential victim

The nurse in the emergency department is assisting in caring for a young female victim of sexual assault. The client's physical assessment is complete, and physical evidence has been collected. The nurse notes that the client is withdrawn, confused, and at times physically immobile. Which interpretation should the nurse make of these behaviors?

They are expected reactions to a devastating event.

A client has reported that crying spells have been a major problem over the past several weeks and that the doctor said depression is probably the reason. The nurse observes that the client is sitting slumped in the chair, and the clothes that the client is wearing do not fit well. The nurse interprets that further data collection should focus on which assessment?

Weight loss

The mother of 6-year-old twins says to the nurse, "My mother-in-law doesn't think our children should come to the funeral service for their grandfather. What do you advise?" Which response made by the nurse would be most appropriate?

What do you and your husband believe....

The spouse of a client admitted to the hospital for alcohol withdrawal says to the nurse, "I should get out of this bad situation." The most helpful response by the nurse should be which statement?

What do you find difficult about this situation?

The client presents with signs and symptoms of anxiety. What conversation initiated by the nurse demonstrates an ineffective therapeutic use of self?

What types of dresses do you like wearing?

A furiously angry and aggressive client was put in restraints and was told that the restraints would be removed once the client regained control. The nurse appropriately removes the restraints when which action occurs?

When no acts of aggression are observed within 1 hour after release of two extremity restraints

Which self-reflective question is directed towards understanding the fundamental basis of personal attitudes about people of diverse cultures?

While a child, what attitudes did your family express about other cultures?

SE of CNS stimulants for ADHD

anorexia - monitor height and weight - give medication just before or at meals stimulation - give last dose before 1600 - avoid caffeine cardiovascular - monitor ECG

ADHD med: SNRI

atomoxetine (Strattera) Is not a CNS stimulant

With tricyclic antidepressants (Amitripyline) monitor clients who have a seizure disorder because the seizure _____________ is decreased.

threshold

The nurse maintains a therapeutic relationship until the client is successfully transferred to another facility. What is the nurse demonstrating?

transitional relationship model

ETOH withdrawal s/s:

withdrawing of CNS depressant - tremor, restlessness, insomnia, irritability - inc HR, inc BP, inc RR, inc temperature - N/V, diaphoresis - hallucinations - seizures

An intoxicated client is brought to the emergency department by local police. The client is told that the primary health care provider (PHCP) will be in to see the client in about 30 minutes. The client becomes very loud and offensive and wants to be seen by the PHCP immediately. The nurse assisting to care for the client should take which appropriate nursing intervention?

Offer to take the client to an examination room until he or she can be treated.

The psychiatric nurse is greeted by a neighbor in a local grocery store. The neighbor says to the nurse, "How is Carol doing? She is my best friend and is seen at your clinic every week." Which is the appropriate nursing response?

I can't tell you that (or similar wording)

A client explains feelings of sadness and loss after the death of a close friend. Which nursing statement best demonstrates empathy?

I see this is hard for you. How can I help?"

A psychiatric-mental health client informs the nurse that a tornado that hit a neighboring town was the client's fault because the client dislikes a neighbor. This disturbance of thought content is known as what?

Ideas of reference

The nurse is caring for a client with seasonal affective disorder (SAD). Which type of therapy is considered a first-line treatment for this disorder?

Light therapy

When assessing a client who has been referred to the outpatient mental health clinic with symptoms of depression, the psychiatric nurse should closely observe the client's affect and which assessment component?

Physical appearance

The parents of a teenager diagnosed with anorexia nervosa ask the nurse what part they can play during the long recovery period. The nurse accurately relates that which actions should the parents take?

Planning a non-food related activity

The nurse is caring for a client recently diagnosed with a psychiatric illness. Which key areas should the nurse focus on while caring for this client? Select all that apply.

Preventing deterioration of mental status Promoting mental health Promoting physical health

The nurse is assigned to a client who is psychotic. The client is pacing, agitated, and using aggressive gestures and rapid speech. The nurse determines which action is the immediate priority of care?

Provide safety for both the client and other clients on the unit.

The nurse is caring for a client with a diagnosis of depression. The nurse monitors for signs of constipation and urinary retention, knowing that these problems are likely caused by which situation?

Psychomotor retardation and side effects of medication

A client is found to have rape-trauma syndrome. The nurse plans care for the client, knowing that rape-trauma syndrome is a condition that involves which?

Reexperiencing recollections of the trauma

Considering the nature of its content, which areas may be the most uncomfortable or difficult for the nurse to assess?

Sexuality

alpha 2 agonists SEs

Slowing down of systems: - CNS sedation - bradycardia, hypoTN - weight gain - GI upset

The nurse is reviewing the health care record of a client admitted to the psychiatric unit. The nurse notes that the admission nurse has documented that the client is experiencing anxiety as a result of a situational crisis. The nurse should determine that this type of crisis could be caused by which event?

The death of a loved one

The nursing student is asked to identify the characteristics of bulimia nervosa. Which characteristic if identified by the student indicates a need to further research the disorder?

Body weight well below ideal range

benzo withdrawal s/s:

CNS excitation, from withdrawing the CNS depressant: - anxiety - insomnia - diaphoresis - tremor - lightheaded - seizures this is why they're supposed to be *short term* if longer term, *taper off*

A client tells the mental health nurse that the client is taking a sewing class to cope with the client's son's move to another state. The use of this adaptive coping skill is an example of which aspect in the therapeutic relationship?

Client self-exploration

The nurse is preparing a psychosocial assessment for use with clients with various mental health conditions. For which group of clients should the nurse include mostly closed-ended questions?

Clients with adult attention deficit hyperactivity disorder

What is a major problem with APs and what's one way to manage it?

Compliance - pts c SCZ frequently stop taking meds r/t SEs, access, expense, etc. Depot formulas - monthly injections (however, safety concerns c these)

The nurse enters a client's room, and the client immediately demands to be released from the hospital. During review of the client's record, the nurse notes that the client was admitted 2 days ago for the treatment of an anxiety disorder and that the admission was a voluntary one. The nurse reports the findings to the registered nurse (RN) and expects that the RN will take which action?

Contact the primary health care provider (PHCP).

A nurse is speaking to a client who attempted suicide. The client says, "It is my dream to become a doctor, and I failed the entrance exam. I am so mad at myself." Which statement indicates genuine interest by the nurse?

You must have been really upset

The nurse is having a conversation with a depressed client in an inpatient psychiatric unit. The client says to the nurse, "Things would be so much better for everyone if I just wasn't around." Which response by the nurse would be appropriate at this time?

You sound very unhappy. Are you thinking of harming yourself?

During a conversation with a depressed client on a psychiatric unit, the client says to the nurse, "My family would be better off without me." The nurse should make which therapeutic response to the client?

You sound very upset. Are you thinking...

A nurse notices that a neighbor has been admitted to an inpatient psychiatric unit. The nurse understands that the nurse may not discuss this with any of the nurse's family or neighbors, because doing so would breach the client's confidentiality. Confidentiality is a component of which element of the therapeutic relationship?

Trust

A nursing student is working with a client who has a history of abusing alcohol. Although the nurse has an aversive feeling toward people who abuse alcohol, the nurse feels that the client is worthy of respect and attention regardless of the nurse's own personal feelings. Which correctly describes the nurse's response to the client?

Unconditional positive regard

A client is diagnosed with schizophrenia. The nurse is asked to assist in preparing a nursing care plan for the client. Which is important for the nurse to understand when planning?

Until the client's thinking is cleared, the nurse may need to assist the client with grooming and nutrition.

A client with delirium becomes agitated and confused at night. The best initial intervention by the nurse is which action?

Use a night light and turn off the television.

A client diagnosed with delirium becomes disoriented and confused at night. Which intervention should the nurse implement initially?

Use an indirect light source and turn off the television.

The nurse is assisting in developing a plan of care for a client with a psychotic disorder who is experiencing altered thought processes. On review of the client's record, the nurse notes documentation that the client believes that the food is being poisoned. The nurse plans to use which communication technique when developing strategies that will promote adequate nutrition and encourage the client to discuss feelings?

Use open-ended questions and silence.

A client who is suicidal tells the nurse, "All I want to do is end it all." Which is the appropriate nursing response?

What do you mean by that?

s/s serotonin syndrome

fever diaphoresis ataxia hyperreflexia, restless confusion hallucinations

why are 3rd gen APs preferred to others?

few EPS/TD lower risk of ACh *lower risk of weight gain*

SSRIs/SNRIs should be used cautiously with which condition?

hepatic impairment (including ETOH dependence)

food to avoid c guanfacine

high fat - increases guanfacine absorption

_______ crisis can occur with MAOIs when there is an intake of dietary tyramine (aged foods, cheeses, fava beans, wine, avocado)

hypertensive

what time of day to take SSRIs

in morning d/t CNS excitation

Clients will have an __________ appetite with tricyclic antidepressants (amitripyline). Instruct the clients to weight weekly and encourage good nutrition/exercise.

increased

Lithium levels PER ATI

initial stabilizing: 0.8-1.4 maintenance: 0.4-1.0 toxicity starts >1.5

The manager schedules a staff nurse to attend a motivational interviewing training session. Which nurse behavior caused the manager to make this decision?

interrupting the client minimizing the client's concerns arguing with a client over agreed upon plans

benzo with short duration?

lorazepam (Ativan)

medication given for hypertensive crisis r/t MAOIs?

phentolamine (rapid alpha blocker) nifedipine (CCB)

Potential serious AEs of trazodone?

priapism cardiac

Cardiac concern with several atypical APs?

prolonged QT interval (can lead to torsades)

Long-term lithium can cause problems with this organ?

thyroid - can cause goiter and hypothyroid

Sedation is an adverse effect usually diminishes over time when taking__________________.

tricyclic antidepressants (Amitripyline)

With ________________ give no more than 1 week supply of medication to client who are acutely ill due to the high risk of lethality with overdose.

tricyclic antidepressants (Amitripyline)

Advise the client to take _________________ at bedtime to minimize daytime sleepiness and to promote sleep.

tricyclic antidepressants (amitripyline/Elavil)

take benzos with food or on empty stomach?

with food - can cause N/V

medication used for EPS (inc dystonia) r/t antipsychotic meds?

- benztropine (Cogentin) - benzodiazepines (usually if Cogentin isn't working) - anticholinergic antihistamines (Benadryl/diphenhydramine, Phenergan/promethazine)

Which client statement indicates the most insight into his or her issue with auditory hallucinations?

"The voices aren't real but it's hard to ignore them."

The nurse is collecting data on a client with the diagnosis of anorexia nervosa. Which findings are indicative of anorexia nervosa? Select all that apply.

-A high achiever -Personality changes -Lanugo over the back and extremities

A client who has terminal cancer has been experiencing a significant increase in pain. However, today the client is no longer complaining of pain but is quiet and isolative. Which types of therapeutic communication should the nurse employ? Select all that apply.

-Sit by client's bed holding his or her hand. -Reminisce with the client and share a humorous client enjoys -The nurse asks: "What can I do... -The nurse asks: "I noticed you grimacing... -The nurse states: "It must be very frustrating...

Which clinical situation provides an example of transference?

A female client with a history of sexual abuse exhibits a profound mistrust of male caregivers.

The nurse employed in a mental health unit of a hospital is the leader of a group psychotherapy session. What is the nurse's role during the termination stage of group development? 1. Acknowledging that the group has identified goals 2. Encouraging the accomplishment of the group's work 3. Acknowledging the contributions of each group member 4. Encouraging members to become acquainted with one another

3. Acknowledging the contributions of each group member Rationale: In the termination stage, the group leader's task is to acknowledge the contributions of each member and the experience of the group as a whole. In this stage, the group members prepare for separation and assist each other to prepare for the future. Acknowledging that the group has identified goals and encouraging group bonding both occur during the initial stage. Encouraging accomplishment of the group's work is appropriate during the working stage.

How to screen for EPS?

Abnormal Involuntary Movement Scale (AIMS) - look @ sticking tongue out

The nurse is preparing to perform a psychosocial assessment of the client. Which describes the most effective approach taken by the nurse?

Accepting Objective

naltrexone is used for?

ETOH withdrawal opioid withdrawal

A client has been brought to the emergency department after attempting to commit suicide by hanging. The nurse should take which nursing action first?

Examine the neck area and assess the airway.

When caring for a client who has been raped, which intervention should the nurse implement during the examination?

Explaining procedures to be completed and why the procedures are necessary

A client experiencing a severe major depressive episode is unable to address activities of daily living. Which is the appropriate nursing intervention?

Feed, bathe, and dress the client as needed until the client can perform these activities independently.

Which is not involved in empathy?

Feeling the same emotions that the client is feeling at a given time

*early* lithium toxicity s/s

GI upset (N/V/D) worsening tremor LOC changes: slurred speech, lethargy

A nurse understands that giving positive regard to the client helps in building trust for the nurse. Which actions are appropriate while conveying positive regard?

The nurse should address the client by name. The nurse should actively listen to the client. The nurse should respond openly to the client.

The nurse is assessing an older adult client with lower back pain. In the course of assessment, the nurse learns that the client lost a spouse 10 weeks ago. The client laughs inappropriately and states, "My spouse just up and left me!" Which is the nurse's best response?

The nurse should recognize the incongruity between content and behavior and find ways of exploring further.

A nurse is reviewing material about assessing mental status. The nurse demonstrates a competent understanding of this assessment by identifying which area as a component of cognition?

abstract reasoning

What is the big bad concern with clozapine

agranulocytosis monitor WBCs

benzo with highest risk of addiction?

alprazolam (Xanax)

After meeting with a client experiencing extreme anxiety the nurse notes personal feelings of nervousness. Which action should the nurse take first to address these feelings?

analyze the source of the feelings

When to administer trazodone?

bedtime - often used for sleep

Any information is only shared with other professionals involved in your care."

beginning the termination process

SSRIs are a first line treatment for __________.

depression

A client who excessively uses alcohol and who is motivated to stop tells the nurse, "I know that there is a medication that can help people like me quit drinking." Which medication should the nurse explain is available for this purpose?

Disulfiram

The nurse is collecting data from a newly admitted client recently diagnosed with borderline personality disorder. Which data provided by the client best supports the nurse's concern that the client is not using effective coping skills?

Driving under the influence (DUI) conviction resulted in a 1-year suspended license

Which client behavior would the nurse document as being an automatism?

Drumming one's fingers on the table top (An automatism is a repeated purposeless behaviors often indicative of anxiety, such as drumming fingers, twisting locks of hair, or tapping the foot)

While providing care to a client with psychosis, the psychiatric nurse uses communication initially for which reason?

Eliciting the client's cooperation through the establishment of trust

While interviewing a client, the nurse puts herself into the client's situation and tries to imagine what it would be like and how it would feel. The nurse is demonstrating what?

Empathy

A nurse is caring for a client with anxiety disorder. The nurse knows that the client will have dyspnea and tachycardia if she has an anxiety attack. According to the Carper's patterns of nursing knowledge, which pattern of knowing is this indicative of?

Empirical knowing

A manic client announces to everyone in the dayroom that a stripper is coming to perform that evening. When the psychiatric nurse's aide firmly states that the client's behavior is not appropriate, the manic client becomes verbally abusive and threatens physical violence to the nurse's aide. Based on the analysis of this situation, the nurse determines that the appropriate action should be which intervention?

Escort the manic client to his or her room.

A nurse has been working for 15 hours continuously without a break. The nurse administrator insists that the nurse should go home and sleep. According to the Carper's patterns of nursing knowledge, which pattern of knowing is this indicative of?

Ethical knowing

Which is the most effective way in which the nurse can assess the progress of a client's mental status based on the expected outcome of the therapeutic plan?

Evaluation

A nurse has been asked to complete a mental status examination of a psychiatric-mental health client. Which is a necessary component of this assessment?

Evaluation of insight and judgment

A female client with anorexia nervosa is a member of a support group. The client has verbalized that she would like to buy some new clothes, but her finances are limited. Group members have brought some used clothes for the client to replace her old clothes. The client believes that the new clothes were much too tight, so she has reduced her calorie intake to 800 calories daily. The nurse identifies this behavior as which finding?

Evidence of the client's altered and distorted body image

The nurse is monitoring a client who abuses alcohol for signs of alcohol withdrawal delirium. The nurse should monitor for which symptoms?

Hypertension, disorientation, hallucinations

A client with moderate depression who was admitted to the mental health unit 2 days ago suddenly begins smiling and reporting that the crisis is over. The client says to the nurse, "I'm finally cured." The nurse interprets this behavior as a cue to modify the treatment plan by taking which action?

Increasing the level of suicide precautions

Which is an inaccurate statement regarding a preconception?

It enables the nurse to get an accurate picture of the client's problems.

The nurse receives a telephone call from a male client who states that he wants to kill himself and has a bottle of sleeping pills in front of him. Which would be the best response by the nurse?

Keep the client talking and signal to another staff member to send help to the client.

A client who has developed paralysis of the lower extremities is admitted to the hospital. The client shares information with the nurse regarding a severe emotional trauma that occurred 6 weeks ago. The nurse develops a plan of care, knowing which action is the priority?

Look for organic causes of the paralysis.

A client tells the nurse that he is feeling out of control. The nurse observes that the client is pacing back and forth. Which approach by the nurse is appropriate to maintain a safe environment?

Move the client to a quiet room and talk about his feelings.

When interacting with a client for the first time, which information would be appropriate for the nurse to disclose?

Name Level of education Reason for being on the unit

An emergency department nurse is caring for an older client who may have been physically abused by her son. In planning care for the client, which is the priority nursing action?

Notify the social worker to...

The day nurses in a psychiatric unit are receiving report from the night shift. During report, a client approaches the nurses' station, becomes very loud and angry, and demands to be seen by the primary health care provider immediately. Which nursing intervention is appropriate?

Offer to assist the client to an examination room until the primary health care provider is notified.

After 5 days in the psychiatric unit, a manic client is able to tolerate short periods in the dayroom. The nurse overhears the client telling another client that he is a journalist posing as a client in order to write an article for a magazine. Which response is the nurse's best action?

Privately confront the client with reality.

A client in a manic state emerges from her room. She is topless and is making sexual remarks and gestures toward staff and peers. Which is an appropriate nursing action?

Quietly approach the client, escort her to her room, and assist her in getting dressed.

Which would be considered a "usual or expected" response during the first few sessions?

Rambling due to nervousness

During the termination phase, a client begins to raise old problems that have already been resolved. Which would be appropriate nursing responses? Select all that apply.

Reassure the client that they already covered these issues. Review with the client the learned methods to control the problems.

Discontinue MAOIs 14 days prior to starting an _________.

SSRI (or before having surgery)

CNS stimulation such as insomnia and agitation can occur when taking ________ for depression.

SSRIs

The nurse is caring for a client diagnosed with catatonic stupor. The client is lying on the bed, with the body pulled into a fetal position. Which is the appropriate nursing intervention?

Sit beside the client in silence and verbalize occasional open-ended questions.

The nurse is caring for a client who says, "I don't want you to touch me. I'll take care of myself!" The nurse should make which therapeutic response to the client?

Sounds like you're feeling pretty troubled...

A client is admitted to the psychiatric nursing unit. When collecting data from the client, the nurse notes that the client was admitted on an involuntary status. Based on this type of admission, which would the nurse expect to note?

The client presents a harm to self.

A client is being prepared for electroconvulsive therapy (ECT). The nurse's plan of care for the night before ECT treatment should include which intervention?

The client shampoos and dries the hair, freeing it of all hair spray and creams.

The nurse is assisting with the data collection on a client admitted to the psychiatric unit. After review of the obtained data, the nurse should identify which as a priority concern?

The client's report of self-destructive thoughts

Avoiding which outcome is the primary reason for establishing professional boundaries with clients

The loss of therapeutic effectiveness

Avoiding which outcome is the primary reason for establishing professional boundaries with clients?

The loss of therapeutic effectiveness

A client expresses worry about the client's child's aggressive behavior. The nurse says "You are in a very challenging situation. Your child's aggressive behavior is very stressful for you, is this correct?" What does this nurse's statement indicate?

The nurse is empathizing with the client.

Which is a nurse's primary tool for treating clients with mental disorders?

The therapeutic use of self

Which is one of the most common reasons clients are often concerned about confidentiality of treatment for mental health problems?

They are worried about the opinions of people who know them outside the hospital, due to shame produced by societal views of mental illness.

The psychiatric nurse recognizes that excessive social communication with a client is to be avoided primarily due to which reason?

To prevent the client from viewing the nurse as a friend rather than health care provider

A nurse is caring for a client with hemiplegia who has been depressed. The client tells the nurse, "I don't feel I would ever be independent again. I would be a burden to everybody in my house." The nurse responds by stating, "Your family misses you a lot and wants you home as soon as possible. The rehab team is very confident about your progress." Which phase of nurse-client relationship is occurring?

Working

During which phase of the nurse-client relationship does the client identify and explore specific problems?

Working

The client tells the nurse, "I am regularly doing my sitting breathing exercises. Why do I still feel breathless while walking?" The nurse replies, "Sitting breathing exercises alone may not achieve the desired effects. You also should perform daily deep breathing exercises while walking. This should help you to reduce breathlessness while walking." According to Peplau's model, the nurse and client are in which phase?

Working

The nurse is caring for a client who has a history of opioid abuse and is monitoring the client for signs of withdrawal. Which manifestations are specifically associated with withdrawal from opioids?

Yawning, irritability, diaphoresis, cramps, and diarrhea

The use of TCAs with alcohol, benzodizepines, opioids, and _____________ can result in additive CNS depression.

antihistamines

MAOIs therapeutic uses

depression bulimia nervosa first-line treatment for atypical depression

Therapeutic uses for tricyclic antidepressants include ______________, neuropathic pain, fibromyalgia, anxiety disorders, insomnia, and bipolar disorder.

depressive disorders

ADHD meds: TCAs

desipramine imipramine clomipramine same SEs apply as c TCAs for other uses

The nurse and client are entering the orientation phase of a relationship. Which is the goal for the client during this phase?

develop a sense of trust in the nurse

A psychiatric nurse tells a client that the nurse will return in 15 minutes to talk with the client. The nurse goes to a meeting that runs overtime and returns in an hour, apologizing for being late. This behavior may have an impact between the nurse and the client in the area of ..

establishing trust in the introductory phase of the relationship.

SEs seen at *therapeutic* lithium levels

fine tremor (can use BBs to treat this) polyuria weight gain

antidote to benzos

flumazenil (Romazicon)

medication for intermittent explosive disorder

fluoxetine

ADHD meds: alpha 2 agonists

guanfacine clonidine

Termination takes place during the resolution phase of a nurse-client relationship. During the termination process, a client brings up resolved problems and presents them as new issues to work toward. The nurse interprets the client's action as indicating what? The client:

is attempting to prolong the nurse-client relationship.

First line med for bipolar depression?

lamotrigine (Lamictal)

*advanced* lithium toxicity s/s

neuro: confusion, sedation, incoordination *coarse* tremor more GI distress

methadone is used for?

opioid dependence - it's also an opioid analgesic - and yes, it's Rx'ed for pain, so don't make assumptions if you see a pt who is taking it! - doesn't produce euphoria - but prevents withdrawal - and allows normal daily activities - longer half-life means fewer peaks/troughs, less likelihood of OD

buprenorphine is used for?

opioid dependence - withdrawal/maintenance

MAOIs can cause _________ so monitor blood pressure and HR for orthostatic changes.

orthostatic hypotension

APs in children (not for SCZ) used for?

risperidone (Risperdal) olanzapine (Zyprexa) pervasive developmental disorders - including ASD conduct disorder OCD same SEs apply as with APs for other uses

why are 2nd gen APs preferred to 1st?

work on positive *and* negative symptoms few EPS, few ACh less relapse

The nurse is caring for a client who is suspected of being dependent on drugs. Which question should be appropriate for the nurse to ask when collecting data from the client regarding drug abuse?

"How much do you use and what effect does it have on you?"

The nurse working the evening shift is assisting clients in getting ready to go to sleep. A client diagnosed with obsessive-compulsive disorder (OCD) becomes upset and agitated and asks the nurse to sit down and talk. Which response by the nurse would be best at this time?

"I can see that you're upset. I'm willing to listen."

The nurse is caring for a client with schizophrenia. The client tells the nurse, "My dead mother is calling me, I will finally be with her tonight. Please do not tell anyone." What is the most appropriate nursing response?

"I cannot keep this a secret. I will ensure that the staff helps keep you safe."

The nurse is providing care for a client admitted to the hospital with a diagnosis of anxiety disorder. The nurse is talking with the client, and the client says, "I have a secret that I want to tell you. You won't tell anyone about it, will you?" Which is the appropriate nursing response?

"I cannot promise to keep a secret."

Which client statement best demonstrates a healthy relationship with family?

"I feel better after I visit with my Mom."

The nurse determines that the wife of an alcoholic client is benefiting from attending an Al-Anon group when the nurse hears the wife make which statement?

"I no longer feel that I deserve the beatings my husband inflicts on me."

The nurse is caring for an older depressed client whose son was killed in an armed robbery after murdering two people. The client says, "I don't know what I did wrong. His dad died a hero in Vietnam when he was only 2 years old, but he's had everything. When he threw the cat up against the wall to see if it landed on its feet and stole money from me and denied it, his sister covered for him." The nurse plans to make which therapeutic response to the client?

"It seems as if you or your daughter feel regret?"

A woman whose husband died 2 months ago says to the visiting nurse, "My daughter came over yesterday to help me move my husband's things out of our bedroom, and I was so angry with her for moving his slippers from where he always kept them under his side of our bed. She doesn't know how much I'm hurting." Which statement by the nurse would be therapeutic?

"It's okay to grieve and be angry with your daughter and anyone else for a time."

The nurse has been caring for a client with a diagnosis of depression. The client says to the nurse, "I wish you would just be my friend." The appropriate response by the nurse is which?

"Our relationship is a therapeutic and a helping one."

Which statement by the nurse demonstrates acceptance to the client who has made a sexually inappropriate comment?

"Our relationship is one of a professional nature."

The nurse is collecting data on a client who is actively hallucinating. Which nursing statement would be therapeutic at this time?

"Sometimes people hear things or voices others can't hear."

The nurse informs a client with an eating disorder about group meetings with Overeaters Anonymous. Which statement by the client indicates the need for further teaching about this self-help group?

"The leader of this self-help group is the nurse or psychiatrist."

During assessment of a client with schizophrenia, the nurse notes the client has ideas of reference. Which statement of the client would have led the nurse to conclude this?

"The news of the terrorist attack is directed to me. The terrorists are trying to warn me."

The nurse working in a detoxification unit is admitting a client for alcohol withdrawal. The client's spouse states, "I don't know why I don't get out of this rotten situation." Which would be a therapeutic response by the nurse?

"What aspects of this situation are the most difficult for you?"

A client who is experiencing suicidal thoughts says to the nurse, "It just doesn't seem to be worth it anymore. Why not just end it all?" Which initial nursing response is appropriate?

"What do you mean by that?"

The nurse is preparing to assess a client's remote memory. Which questions would be most appropriate for the nurse to ask?

"When did you get your first job?"

A client who is diagnosed with pedophilia and recently has been paroled as a sex offender says, "I'm in treatment and I have served my time. Now this group has posters all over the neighborhood with my photograph and details of my crime." Which is an appropriate response by the nurse?

"You understand that people fear for their children, but you're feeling unfairly treated?"

buspirone: how is it different from benzos?

*scheduled* not PRN *slow onset - weeks to peak* not fast acting *does not sedate* unlike BZDs but can cause dizziness

Pt teaching for any anticholinergic meds (inc TCAs)

- dry mouth: sugarless gum/candy/mints, oral rinses (like Biotene), frequent sips of water - photophobia: wear sunglasses outdoors - OH: change positions slowly - constipation: increase fluids and fiber, exercise - urinary hesitancy: urinate right before taking med

*super bad* lithium toxicity s/s

- extreme polyuria, dilute urine - jerky movements, ataxia, seizure - tinnitus, blurry vision - stupor leading to coma and death

atomoxetine concerns

- weight, appetite - take c food PRN GI upset - risk of SI - hepatotoxicity - last dose before 1600

The nurse caring for a client with schizophrenia prepares to document which signs/symptoms exhibited by the client as negative? Select all that apply.

-Avolition -Anergia

The nurse is caring for a client with an eating disorder and knows that which signs/symptoms indicate that the client is dealing with anorexia nervosa? Select all that apply.

-Lanugo -Amenorrhea

When considering where to conduct a psychosocial assessment, the nurse can effectively interview which client in the unit's conference room?

. the anxious client

The nurse is preparing a client for electroconvulsive therapy (ECT), which is scheduled for the next morning. Which interventions would be included in the preprocedural plan? Select all that apply. 1. Obtain an informed consent. 2. Have the client void before the procedure. 3. Remove dentures and contact lenses before the procedure. 4. Withhold food and fluids for 6 hours before the treatment. 5. Administer tap water enemas on the evening before the procedure.

1, 2, 3, 4 Rationale: Enemas are not a component of the pretreatment care for a client scheduled for ECT. Options 1, 2, 3, and 4 are a part of the pretreatment plan. Additionally, the nurse should teach the client and family what to expect with ECT and allow the client to discuss his or her feelings regarding the procedure.

Which interventions are most appropriate for caring for a client in alcohol withdrawal? Select all that apply. 1. Monitor vital signs. 2. Maintain NPO status. 3. Provide a safe environment. 4. Address hallucinations therapeutically. 5. Provide stimulation in the environment. 6. Provide reality orientation as appropriate.

1, 3, 4, 6 Rationale: When the client is experiencing withdrawal from alcohol, the priority for care is to prevent the client from harming self or others. The nurse would provide a low-stimulation environment to maintain the client in as calm a state as possible. The nurse would monitor the vital signs closely and report abnormal findings. The nurse would reorient the client to reality frequently and would address hallucinations therapeutically. Adequate nutritional and fluid intake need to be maintained.

A client is unwilling to go out of the house for fear of "making a fool of myself in public." Because of this fear, the client remains homebound. Based on these data, which mental health disorder is the client experiencing? 1. Agoraphobia 2. Social phobia 3. Claustrophobia 4. Hypochondriasis

2. Social phobia Rationale: Social phobia is a fear of situations in which one might be embarrassed or criticized, such as the fear of speaking, performing, or eating in public. The person fears making a fool of oneself. Agoraphobia is a fear of open spaces and the fear of being trapped in a situation from which there may not be an escape. Claustrophobia is a fear of closed places. Clients with hypochondriacal symptoms focus their anxiety on physical complaints and are preoccupied with their health.

The nurse is reviewing the assessment data of a client admitted to the mental health unit. The nurse notes that the admission nurse documented that the client is experiencing anxiety as a result of a situational crisis. The nurse determines that this type of crisis could be caused by which event? 1. Witnessing a murder 2. The death of a loved one 3. A fire that destroyed the client's home 4. A recent rape episode experienced by the client

2. The death of a loved one Rationale: A situational crisis arises from external rather than internal sources. External situations that could precipitate a crisis include loss of or change of a job, the death of a loved one, abortion, change in financial status, divorce, addition of new family members, pregnancy, and severe illness. Options 1, 3, and 4 identify adventitious crises. An adventitious crisis refers to a crisis of disaster; it is unplanned or accidental.

A client says to the nurse, "The federal guards were sent to kill me." What is the best nursing response to the client's concern? 1. "I don't believe this is true." 2. "The guards are not out to kill you." 3. "Do you feel afraid that people are trying to hurt you?" 4. "What makes you think the guards were sent to hurt you?"

3. "Do you feel afraid that people are trying to hurt you?" Rationale: It is most therapeutic for the nurse to empathize with the client's experience. The remaining options lack this connection with the client. Disagreeing with delusions may make the client more defensive, and the client may cling to the delusions even more. Encouraging discussion regarding the delusion is inappropriate.

Which describes the primary focus of milieu therapy? 1. A form of behavior modification therapy 2. A cognitive approach to changing behavior 3. A living, learning, or working environment 4. A behavioral approach to changing behavior

3. A living, learning, or working environment Rationale: Milieu therapy, or "therapeutic community," has as its focus a living, learning, or working environment. Such therapy may be based on numerous therapeutic modalities ranging from structured behavioral therapy to spontaneous, humanistically oriented approaches. Although milieu therapy may include behavioral approaches, the correct option describes its primary focus.

A manic client begins to make sexual advance towards visitors in the dayroom. When the nurse firmly states that this is inappropriate and will not be allowed, the client becomes verbally abusive and threatens physical violence to the nurse. Based on the analysis of this situation, which intervention should the nurse implement? 1. Place the client in seclusion for 30 minutes. 2. Tell the client that the behavior is inappropriate. 3. Escort the client to their room, with the assistance of other staff. 4. Tell the client that their telephone privileges are revoked for 24 hours.

3. Escort the client to their room, with the assistance of other staff. Rationale: The client is at risk for injury to self and others and should be escorted out of the dayroom. Seclusion is premature in this situation. Telling the client that the behavior is inappropriate has already been attempted by the nurse. Denying privileges may increase the agitation that already exists in this client.

The nurse is caring for a client diagnosed with catatonic stupor who is lying on the bed in a fetal position. What is the most appropriate nursing intervention? 1. Ask direct questions to encourage talking. 2. Leave the client alone so as to minimize external stimuli. 3. Sit beside the client in silence with occasional open-ended questions. 4. Take the client into the dayroom with other clients so that they can help watch him.

3. Sit beside the client in silence with occasional open-ended questions. Rationale: Clients who are withdrawn may be immobile and mute and may require consistent, repeated approaches. Communication with withdrawn clients requires much patience from the nurse. Interventions include the establishment of interpersonal contact. The nurse facilitates communication with the client by sitting in silence, asking open-ended questions rather than direct questions, and pausing to provide opportunities for the client to respond. While overstimulation is not appropriate, there is no therapeutic value in ignoring the client. The client's safety is not the responsibility of other clients.

A nursing instructor teaches a group of nursing students about violence in the family. Which statement by a student indicates a need for further teaching? 1. "Abusers use fear and intimidation." 2. "Abusers usually have poor self-esteem." 3. "Abusers often are jealous or self-centered." 4. "Abuse occurs more often in low-income families."

4. "Abuse occurs more often in low-income families." Rationale: Personal characteristics of abusers include low self-esteem, immaturity, dependence, insecurity, and jealousy. Abusers often use fear and intimidation to the point at which their victims will do anything just to avoid further abuse. The statement that abuse occurs more often in lower socioeconomic groups is incorrect.

A client with a diagnosis of depression has been meeting with the mental health nurse for therapy sessions for the past 6 weeks. During the session the client says to the nurse, "I lost my job this week, and I'm going to be evicted from my apartment if I can't pay my bill. The only person that I have is my daughter, but I don't want to burden her with my problems." Which response by the nurse would be therapeutic? 1. "Why did you lose your job?" 2. "There are homeless shelters available, and we will get you into one if you are evicted from your apartment." 3. "If you get evicted from your apartment, we will commit you to the hospital, so you will have a place to eat and sleep." 4. "Let's talk about contacting your daughter. Wouldn't you want to know if your daughter was having difficulty and try to help her if you could?"

4. "Let's talk about contacting your daughter. Wouldn't you want to know if your daughter was having difficulty and try to help her if you could?" Rationale: The therapeutic communication technique is clarification that attempts to put vague ideas into words. It helps the client to view the explicit correlation between the client's feelings and actions. Asking why a client lost a job is not directly related to the client's feelings and concerns. Offering to provide a homeless shelter or to commit the client to the hospital does not address the issue at hand and places the client's concerns and feelings on hold.

A client experiencing a great deal of stress and anxiety is being taught to use self-control therapy. Which statement by the client indicates a need for further teaching about the therapy? 1. "This form of therapy can be applied to new situations." 2. "An advantage of this technique is that change is likely to last." 3. "Talking to oneself is a basic component of this form of therapy." 4. "This form of therapy provides a negative reinforcement when the stimulus is produced."

4. "This form of therapy provides a negative reinforcement when the stimulus is produced." Rationale: Negative reinforcement when the stimulus is produced is descriptive of aversion therapy. Options 1, 2, and 3 are characteristics of self-control therapy.

A client with a diagnosis of major depression who has attempted suicide says to the nurse, "I should have died. I've always been a failure. Nothing ever goes right for me." Which response demonstrates therapeutic communication? 1. "You have everything to live for." 2. "Why do you see yourself as a failure?" 3. "Feeling like this is all part of being depressed." 4. "You've been feeling like a failure for a while?"

4. "You've been feeling like a failure for a while?" Rationale: Responding to the feelings expressed by a client is an effective therapeutic communication technique. The correct option is an example of the use of restating. The remaining options block communication because they minimize the client's experience and do not facilitate exploration of the client's expressed feelings. In addition, use of the word "why" is nontherapeutic.

The nurse is monitoring a client with a diagnosis of schizophrenia. The nurse notes that the client's emotional responses to situations occurring throughout the day are incongruent with the tone of the situation. The nurse should document the findings using which description of the client's behavioral response? 1. Flat affect 2. Bizarre affect 3. Blunted affect 4. Inappropriate affect

4. Inappropriate affect Rationale: An inappropriate affect refers to an emotional response to a situation that is incongruent with the tone of the situation. A flat affect is manifested as an immobile facial expression or blank look. A bizarre affect such as grimacing, laughing, and self-directed mumbling is marked when the client is unable to relate logically to the environment. A blunted affect is a minimal emotional response or outward affect that typically does not coincide with the client's inner emotions.

The nurse is monitoring a client who has been placed in restraints because of violent behavior. When should the nurse determine that it will be safe to remove the restraints? 1. Administered medication has taken effect. 2. The client verbalizes the reasons for the violent behavior. 3. The client apologizes and tells the nurse that it will never happen again. 4. No acts of aggression have been observed within 1 hour after the release of two of the extremity restraints.

4. No acts of aggression have been observed within 1 hour after the release of two of the extremity restraints. Rationale: The best indicator that the behavior is controlled is the fact that the client exhibits no signs of aggression after partial release of restraints. Options 1, 2, and 3 do not ensure that the client has controlled the behavior.

The nurse has developed a plan of care for a client diagnosed with anorexia nervosa. Which client problem would the nurse select as the priority in the plan of care? 1. Disrupted appearance because of weight 2. Inability to feed self because of weakness 3. Pain because of an inflamed gastric mucosa 4. Nutritional imbalance because of lack of intake

4. Nutritional imbalance because of lack of intake Rationale: The priority client problem for the client with anorexia nervosa is lack of intake and nutritional imbalance. Although the problems identified in options 1, 2, and 3 may be considerations in the plan of care for the client with anorexia nervosa, nutritional imbalance is the priority.

While being treated, a client is introduced to short periods of exposure to the phobic object while in a relaxed state. What term is used to describe this form of behavior modification? 1. Milieu therapy 2. Aversion therapy 3. Self-control therapy 4. Systematic desensitization

4. Systematic desensitization Rationale: Systematic desensitization is a form of therapy used when the client is introduced to short periods of exposure to the phobic object while in a relaxed state. Exposure is gradually increased until the anxiety about or fear of the object or situation has ceased. Milieu management refers to providing a safe, therapeutic environment and is applicable to not just this scenario. The remaining options are incorrect since they do not involve the intervention described.

Which client is most likely at risk to become a victim of elder abuse?

A 90-year-old woman with advanced Alzheimer's disease

A nurse administrator is observing the behavior of nurses in the hospital. Which behaviors would the nurse administrator consider inappropriate?

A nurse hugging a client who had come in for an initial visit A nurse speaking to a depressed client in a very strict, disciplinarian tone

If the client provides a literal explanation of a proverb and cannot interpret its meaning, which thought process is lacking?

Abstract thinking

A client in a manic state emerges from her room. The client is dressed in a low-cut blouse and a miniskirt. She is not wearing underwear and she proceeds to sit on a male client's lap and begins to make sexual remarks and gestures to the male client. The nurse should take which action?

Approach the client quietly, take her to her room, and assist her in getting dressed.

The nurse learns that a new client is a former significant other and an initial session is scheduled for early in the afternoon. Which action should the nurse take to maintain professional boundaries?

Ask to be reassigned because of having a prior personal relationship with the client.

A client with a diagnosis of bipolar I disorder has been presented with a coping strategy by the therapist that may help the client manage behavior during manic episodes. The client has responded to the therapist's suggestion by saying, "What's the use? I don't ever see this changing." The client's statement is suggestive of a potential problem with what factor that influences communication?

Attitude

A client is admitted to a psychiatric unit for treatment of a psychotic disorder. The client is at the locked exit door and is shouting, "Let me out! There's nothing wrong with me! I don't belong here!" The nurse identifies this behavior as which defense mechanism?

Denial

The manager is reviewing the implementation of the Transitional Relationship Model (TRM) on a care area. Which data should the manager identify that supports the successful implementation of this model?

Discharge of clients 5 days earlier than in the past Number of readmissions down 6% over the last 2 months

The nurse notes that a client with acquired immunodeficiency syndrome (AIDS) appears anxious and is reluctant to ask questions. Which action should the nurse take to best address these observations?

Discuss common fears and questions expressed by other clients with the same diagnosis.

A client who was hospitalized for depression is being prepared by the nurse for discharge. In evaluating the coping strategies learned during hospitalization, the nurse should recognize which statement by the client is an indication that further teaching is needed?

"I know that I won't become depressed again."

The nurse has entered a hospital client's room and asked the client if the client plans to attend the morning's scheduled group life-skills session. Which response should signal the presence of thought blocking to the nurse?

"I might. I'll give it some..."

A client is admitted to the in-patient unit and is being considered for electroconvulsive therapy (ECT). The client appears calm, but the family is hypervigilant and anxious. The client's mother begins to cry and states, "My child's brain will be destroyed. How can the doctor do this?" The nurse should make which therapeutic response?

"It sounds as though you have some concerns about the ECT procedure. Why don't we sit down together and discuss any concerns you may have?"

The nurse is caring for a hospitalized client who is suspicious and guarded. The client tells the nurse that the client does not want anyone to tell the family about the client's condition. What is the nurse's best response when the family calls the hospital unit to inquire about the client's condition?

"You are welcome to share any information that you think would be helpful."

The nurse on a behavioral health unit is having a therapeutic discussion with a client and recognizes that which communication techniques would be nontherapeutic? Select all that apply.

-Minimizing -Changing -Asking

The nurse is caring for a client in the acute manic stage of bipolar disorder and plans to use which interventions to assist in maintaining a safe environment? Select all that apply.

-Provide -Decrease -Restrict

The nurse is preparing to perform an admission assessment on a client with a diagnosis of bulimia nervosa. Which assessment findings does the nurse expect to note? Select all that apply. 1. Dental decay 2. Moist oily skin 3. Loss of tooth enamel 4. Electrolyte imbalances 5. Body weight well below ideal range

1, 3, 4 Rationale: Clients with bulimia nervosa initially may not appear to be physically or emotionally ill. They are often at or slightly below ideal body weight. On further inspection, a client exhibits dental decay and loss of tooth enamel if the client has been inducing vomiting. Electrolyte imbalances are present. Dry, scaly skin (rather than moist, oily skin) is present.

A client being seen in the emergency department immediately after being sexually assaulted appears calm and controlled. The nurse analyzes this behavior as indicating which defense mechanism? 1. Denial 2. Projection 3. Rationalization 4. Intellectualization

1. Denial Rational: Denial is refusal to admit to a painful reality and may be a response by a victim of sexual abuse. In this case the client is not acknowledging the trauma of the assault either verbally or nonverbally. Projection is transferring one's internal feelings, thoughts, and unacceptable ideas and traits to someone else. Rationalization is justifying the unacceptable attributes about oneself. Intellectualization is the excessive use of abstract thinking or generalizations to decrease painful thinking.

The home care nurse is visiting an older client whose spouse died 6 months ago. Which behavior by the client indicates ineffective coping? 1. Neglecting personal grooming 2. Looking at old snapshots of family 3. Participating in a senior citizens' program 4. Visiting their spouse's grave once a month

1. Neglecting personal grooming Rational: Coping mechanisms are behaviors used to decrease stress and anxiety. In response to a death, ineffective coping is manifested by an extreme behavior that in some cases may be harmful to the individual physically or psychologically. The correct option is indicative of a behavior that identifies an ineffective coping behavior in the grieving process.

A client with a diagnosis of anorexia nervosa, who is in a state of starvation, is in a two-bed hospital room. A newly admitted client will be assigned to this client's room. Which client should be an appropriate choice as this client's roommate?

A client receiving diagnostic tests

During a therapy session with a client with paranoid disorder, the client says to the nurse, "You look so nice today. I love how you do your hair, and I love that perfume you're wearing." Which response by the nurse would be therapeutic? 1. "Your comment is inappropriate." 2. "Thank you for noticing. I just bought this new perfume." 3. "My hair has been a mess. I really needed to have it done." 4. "We are not here to discuss how I look or smell. We are here to talk about you."

4. "We are not here to discuss how I look or smell. We are here to talk about you." Rationale: The therapeutic response by the nurse is the one that clarifies the content of the client's statements and directs the client to the purpose of the session. The nurse should confront the client verbally regarding the inappropriate statements and refocus the client back to the issue of the session. Option 1 may be judgmental and may provide an opening for a verbal struggle. Options 2 and 3 are social responses and could be misinterpreted by the client.

The nurse is preparing a client with a history of command hallucinations for discharge by providing instructions on interventions for managing hallucinations and anxiety. Which statement in response to these instructions suggests to the nurse that the client understands the instructions? 1. "My medications aren't likely to make me anxious." 2. "I'll go to support group and talk so that I don't hurt anyone." 3. "It's not likely that I'll get anxious or hear things if I get enough sleep and eat well." 4. "When I begin to hallucinate, I'll call my therapist and talk about what I should do."

4. "When I begin to hallucinate, I'll call my therapist and talk about what I should do." Rationale: The risk for impulsive and aggressive behavior may increase if a client is receiving command hallucinations to harm self or others. If the client is experiencing a hallucination, the nurse should ask the client whether he or she has intentions to hurt him- or herself or others. Talking about auditory hallucinations can interfere with subvocal muscular activity associated with a hallucination. The client statements in the remaining options will aid in wellness, but are not specific interventions for hallucinations, if they occur.

The mental health nurse is caring for a client with a social phobia. The nurse tells the client that a music therapy session has been scheduled as part of the treatment plan. The client tells the nurse that she cannot sing and refuses to attend. What is the appropriate nursing response? 1. "You must go. You have no choice." 2. "Why don't you want to attend? What is the real reason?" 3. "The health care provider has prescribed this therapy for you." 4. "You don't have to sing at the session. You can listen and enjoy the music."

4. "You don't have to sing at the session. You can listen and enjoy the music." Rationale: The correct option encourages the client to socialize and indicates that it is not necessary to sing. Option 2 asks why, and use of this word should be avoided. Options 1 and 3 imply a demand and do not address the client's concern. The correct option is the only one that addresses the client's concern.

The nurse is developing a plan of care for a client in a crisis state. When developing the plan, the nurse should consider which factor? 1. A crisis state indicates that the client has a mental illness. 2. A crisis state indicates that the client has an emotional illness. 3. Presenting symptoms in a crisis situation are similar for all clients experiencing a crisis. 4. A client's response to a crisis is individualized and what constitutes a crisis for one client may not constitute a crisis for another client.

4. A client's response to a crisis is individualized and what constitutes a crisis for one client may not constitute a crisis for another client. Rationale: Although each crisis response can be described in similar terms as far as presenting symptoms are concerned, what constitutes a crisis for one client may not constitute a crisis for another client because each is a unique individual. Being in the crisis state does not mean that the client has a mental or emotional illness.

A client is admitted to the mental health unit with a diagnosis of depression. The nurse should develop a plan of care for the client that includes which intervention? 1. Encouraging quiet reading and writing for the first few days 2. Identification of physical activities that will provide exercise 3. No socializing activities, until the client asks to participate in milieu 4. A structured program of activities in which the client can participate

4. A structured program of activities in which the client can participate Rationale: A client with depression often is withdrawn while experiencing difficulty concentrating, loss of interest or pleasure, low energy, fatigue, and feelings of worthlessness and poor self-esteem. The plan of care needs to provide successful experiences in a stimulating yet structured environment. The remaining options are either too "restrictive" or offer little or no structure and stimulation.

On review of the client's record, the nurse notes that the mental health admission was voluntary. Based on this information, the nurse anticipates which client behavior? 1. Fearfulness regarding treatment measures. 2. Anger and aggressiveness directed toward others. 3. An understanding of the pathology and symptoms of the diagnosis. 4. A willingness to participate in the planning of the care and treatment plan.

4. A willingness to participate in the planning of the care and treatment plan. Rationale: In general, clients seek voluntary admission. If a client seeks voluntary admission, the most likely expectation is that the client will participate in the treatment program since they are actively seeking help. The remaining options are not characteristics of this type of admission. Fearfulness, anger, and aggressiveness are more characteristic of an involuntary admission. Voluntary admission does not guarantee a client's understanding of their illness, only of their desire for help.

The nurse is admitting a client with a diagnosis of posttraumatic stress disorder to the mental health unit. The client is confused and disoriented. During the assessment, what is the nurse's primary goal for this client? 1. Explain the unit rules. 2. Orient the client to the unit. 3. Stabilize the client's psychiatric needs. 4. Accept the client and make the client feel safe.

4. Accept the client and make the client feel safe. Rationale: It is important to make a confused client feel safe. Explaining the unit rules and orientation to the unit are part of any admission process. Stabilizing psychiatric needs is a long-term goal.

During a home visit, the nurse suspects that a young daughter of the client is bulimic. The nurse bases this suspicion on which primary characteristic of bulimia? 1. Refusing to eat and excessive exercising 2. Eating only vegetables and fruits and fasting 3. Hoarding of food and difficulty controlling food intake 4. Eating a lot of food in a short period of time and misuse of laxatives

4. Eating a lot of food in a short period of time and misuse of laxatives Rationale: Eating binges and purging are the characteristic that would be seen in bulimia. Eating only certain types of foods may reflect a preference but does not indicate bulimia. Bulimic persons usually do not refuse to eat; rather, they binge and purge. Hoarding of food may indicate another problem.

A client recently admitted to the hospital in the manic phase of bipolar disorder is dehydrated, unkempt, taking antipsychotic medications, and complaining of abdominal fullness and discomfort. The nurse determines that which intervention is most appropriate for these complaints? 1. Teach self-grooming skills. 2. Reward cleanliness with unit privileges. 3. Monitor the adequacy of the antipsychotic dosage. 4. Encourage frequent fluid intake and a high-fiber diet.

4. Encourage frequent fluid intake and a high-fiber diet. Rationale: Constipation is a common elimination problem with clients in a manic phase of bipolar disorder. Constipation may occur as the result of a combination of factors, including taking antipsychotic medications, suppressing the urge to defecate, and a decreased fluid intake as a result of the manic activity level. The symptoms listed in the question, dehydrated, unkempt, and abdominal fullness and discomfort, in combination with antipsychotic medications, are indicators of constipation. A high-fiber diet and increased fluids can reduce constipation.

When a client is admitted to an inpatient mental health unit with the diagnosis of anorexia nervosa, a cognitive behavioral approach is used as part of the treatment plan. The nurse understands that which is the purpose of this approach? 1. Providing a supportive environment 2. Examining intrapsychic conflicts and past issues 3. Emphasizing social interaction with clients who withdraw 4. Helping the client to examine dysfunctional thoughts and beliefs

4. Helping the client to examine dysfunctional thoughts and beliefs Rationale: Cognitive behavioral therapy is used to help the client identify and examine dysfunctional thoughts and to identify and examine values and beliefs that maintain these thoughts. The remaining options, while therapeutic in certain situations, are not the focus of cognitive behavioral therapy.

The nurse is monitoring a hospitalized client who abuses alcohol. Which findings should alert the nurse to the potential for alcohol withdrawal delirium? 1. Hypotension, ataxia, hunger 2. Stupor, lethargy, muscular rigidity 3. Hypotension, coarse hand tremors, lethargy 4. Hypertension, changes in level of consciousness, hallucinations

4. Hypertension, changes in level of consciousness, hallucinations Rationale: Symptoms associated with alcohol withdrawal delirium typically include anxiety, insomnia, anorexia, hypertension, disorientation, hallucinations, changes in level of consciousness, agitation, fever, and delusions.

The nurse is planning care for a client with bipolar disorder who is experiencing psychomotor agitation. Which activity should the nurse plan for this client? 1. Reading letters and books in a quiet environment 2. Providing an activity such as checkers for the client 3. Involving the client in a card game with other clients on the unit 4. Including the client in a clay-molding class that is scheduled for today

4. Including the client in a clay-molding class that is scheduled for today Rationale: When a client is experiencing psychomotor agitation, it is best to provide activities that involve the use of hands and gross motor movements. Such activities can include volleyball, finger-painting, drawing, and working with clay. These activities provide an appropriate way for the client to discharge motor tension. Reading and simple card games are sedentary activities. Playing checkers requires concentration and more intensive use of thought processes.

The nurse is working with a client who despite making a heroic effort was unable to rescue a neighbor trapped in a house fire. Which client-focused action should the nurse engage in during the working phase of the nurse-client relationship? 1. Exploring the client's ability to function 2. Exploring the client's potential for self-harm 3. Inquiring about the client's perception or appraisal of why the rescue was unsuccessful 4. Inquiring about and examining the client's feelings for any that may block adaptive coping

4. Inquiring about and examining the client's feelings for any that may block adaptive coping Rational: The client must first deal with feelings and negative responses before the client can work through the meaning of the crisis. The correct option pertains directly to the client's feelings and is client-focused. The remaining options do not directly focus on or address the client's feelings.

The nurse assesses a client with the admitting diagnosis of bipolar affective disorder, mania. Which client symptoms require the nurse's immediate action? 1. Incessant talking and sexual innuendoes 2. Grandiose delusions and poor concentration 3. Outlandish behaviors and inappropriate dress 4. Nonstop physical activity and poor nutritional intake

4. Nonstop physical activity and poor nutritional intake Rationale: Mania is a mood characterized by excitement, euphoria, hyperactivity, excessive energy, decreased need for sleep, and impaired ability to concentrate or complete a single train of thought. The client's mood is predominantly elevated, expansive, or irritable. All of the options reflect a client's possible symptoms. The correct option clearly presents a problem, however, that compromises physiological integrity and needs to be addressed immediately.

The nurse is reviewing the record of a client admitted to the mental health unit. The nurse notes documentation that the client experiences flashbacks. What diagnosis should the nurse expect to be documented for this client? 1. Anxiety 2. Agoraphobia 3. Schizophrenia 4. Posttraumatic stress disorder (PTSD)

4. Posttraumatic stress disorder (PTSD) Rationale: The major clinical manifestation associated with PTSD is client experience of flashbacks. Flashbacks are not specifically associated with anxiety, agoraphobia, or schizophrenia.

The nurse is planning a stress management seminar for clients in an ambulatory care setting. Which concept should the nurse plan to include in the content of the seminar? 1. Biofeedback has the advantage of using no equipment at all. 2. Guided imagery is a helpful technique but requires video equipment for its use. 3. Confrontation is a useful method for solving potentially stressful conflicts with others. 4. Progressive muscle relaxation techniques are useful for easing tension from many causes.

4. Progressive muscle relaxation techniques are useful for easing tension from many causes. Rationale: Biofeedback, guided imagery, progressive muscle relaxation, and meditation are techniques that the nurse can teach the client to reduce the physical impact of stress on the body and promote a feeling of self-control. Biofeedback uses electronic equipment, whereas each of the other techniques requires no equipment after it is learned. Confrontation is not a stress management technique; it is a communication technique.

The mental health nurse is talking to a client who has been diagnosed with posttraumatic stress disorder. During the conversation, the nurse notes that the client is exhibiting a paranoid stare and that he begins to pace and fidget. What is the appropriate nursing intervention? 1. Allow the client to pace. 2. Escort the client to a quiet room. 3. Change the conversation to a less threatening subject. 4. Share the observation with the client and help the client to recognize his feelings.

4. Share the observation with the client and help the client to recognize his feelings. Rationale: Sharing observations with the client may help him recognize and acknowledge feelings. Allowing the client to pace may also allow him to get out of control. Moving to a quiet room or changing the subject will not help the client to recognize his behaviors and feelings.

A client's unresolved feelings related to loss would be most likely observed during which phase of the therapeutic nurse-client relationship? 1. Trusting 2. Working 3. Orientation 4. Termination

4. Termination In the termination phase, the relationship comes to a close. Ending treatment sometimes may be traumatic for clients who have come to value the relationship and the help. Because loss is an issue, any unresolved feelings related to loss may resurface during this phase. The remaining options are not specifically associated with this issue of unresolved feelings.

What is the most appropriate nursing action to help manage a manic client who is monopolizing a group therapy session? 1. Ask the client to leave the group for this session only. 2. Refer the client to another group that includes other manic clients. 3. Tell the client to stop monopolizing in a firm but compassionate manner. 4. Thank the client for the input, but inform the client that now others need a chance to contribute.

4. Thank the client for the input, but inform the client that now others need a chance to contribute. Rationale: If a client is monopolizing the group, the nurse must be direct and decisive. The best action is to thank the client and suggest that the client stop talking and try listening to others. Although telling the client to stop monopolizing in a firm but compassionate manner may be a direct response, the correct option is more specific and provides direction for the client. The remaining options are inappropriate since they are not directed towards helping the client in a therapeutic manner.

A homebound client confidentially discusses suicidal plans with the visiting nurse. Based on professional duty to observe confidentiality, which statement best describes the nurse's obligation to the client? 1. The nurse must have the client go to the local mental health center daily for counseling. 2. The nurse must ask the client not to reveal suicidal plans if the information needs to be kept confidential. 3. The nurse cannot tell anyone what the client said and must strictly adhere to the professional duty for confidentiality. 4. The nurse must override the duty to observe confidentiality and notify the client's health care provider (HCP) about the suicidal ideation.

4. The nurse must override the duty to observe confidentiality and notify the client's health care provider (HCP) about the suicidal ideation. Rationale: In this situation, the nurse must override the duty to observe confidentiality and notify the client's HCP about the client's suicidal ideation. Option 1 is incorrect because the client is homebound. Option 2 is incorrect because the nurse has a professional obligation to intervene when a client tells the nurse about ideas or plans to harm himself or herself or others. Option 3 is incorrect because the nurse has a moral obligation to protect the client.

A client has been prescribed disulfiram (Antabuse). Before giving the client the first dose of this medication, what should the psychiatric home health nurse determine? 1. If there is a history of hyperthyroidism 2. When the last full meal was consumed 3. If there is a history of diabetes insipidus 4. When the last alcoholic drink was consumed

4. When the last alcoholic drink was consumed Rationale: Disulfiram is an adjunctive treatment for some clients with chronic alcoholism to assist in maintaining enforced sobriety. Because clients must abstain from alcohol for at least 12 hours before the initial dose, the most important assessment is when the last alcoholic intake was consumed. The medication should be used cautiously in clients with hypothyroidism, diabetes mellitus, epilepsy, cerebral damage, nephritis, and hepatic disease. It is contraindicated in persons with severe heart disease, psychosis, or hypersensitivity to the medication.

A hospitalized client is receiving clozapine (Clozaril) for the treatment of a schizophrenic disorder. The nurse determines that the client may be having an adverse reaction to the medication if abnormalities are noted on which laboratory study? 1. Platelet count 2. Cholesterol level 3. Blood urea nitrogen 4. White blood cell (WBC) count

4. White blood cell (WBC) count Rationale: Clozapine is an antipsychotic medication. Clients taking clozapine can experience hematological adverse effects, including agranulocytosis and mild leukopenia. The WBC count should be assessed before initiation of treatment and should be monitored closely during the use of this medication. The client also should be monitored for signs indicating agranulocytosis, which may include sore throat, malaise, and fever. Options 1, 2, and 3 are incorrect and unrelated to this medication.

The nurse is assisting with creating a plan of care for the client in a crisis state. When developing the plan, the nurse should consider which about a crisis response?

A client's response to a crisis is individualized, and what constitutes a crisis for one person may not constitute a crisis for another person.

A 35-year-old was discharged from care after recovery from depression. The nurse therapist and the client spent many hours working through issues related to the depression. Six months later, the client is admitted again for depression associated with issues similar to those that were previously addressed in the client's therapy. The nurse therapist says to a coworker, "This is unbelievable; we're back at square one again. The client should know better at this point." The nurse's comments reflect what?

A judgmental attitude

During a first meeting a client asks that information shared will not be relayed to immediate family members. Which should the nurse say in response?

Any information is only shared with other professionals involved in your care."

The nurse notes that an older adult client is wearing layers of clothing on a warm, fall day. What would be the priority assessment at this time?

Asking whether the client often feels cold

A client was admitted to a medical unit with acute blindness. Many tests are performed, and there seems to be no organic reason why this client cannot see. The nurse later learns that the client became blind after witnessing a hit-and-run car crash in which a family of three was killed. The nurse suspects that the client may be experiencing which diagnosis?

Conversion disorder

The nurse is monitoring a client with a history of opioid abuse for signs/symptoms of withdrawal. The nurse monitors this client for which signs/symptoms associated with opioid withdrawal?

Depression, high drug craving, fatigue with altered sleep (insomnia or hypersomnia), agitation, and paranoia

When comparing social interactions with therapeutic interactions, the nurse understands that therapeutic interactions do what?

Encourage personal goal setting

Following a group therapy session, a client approaches the nurse and verbalizes a need for seclusion because of uncontrollable feelings. The nurse reports the findings to the registered nurse (RN) and expects that the RN will take which action?

Get a written prescription from the primary health care provider (PHCP) and obtain an informed consent.

A visitor brings a suicidal client a brightly packaged gift. The nurse accompanies the visitor to the client's room and takes which action?

Has the client open the gift with the nurse present

The nurse is assigned to care for a client admitted to the hospital after sustaining an injury from a house fire. The client attempted to save a neighbor involved in the fire, but despite the client's efforts, the neighbor died. Which action should the nurse take to enable the client to work through the meaning of the crisis?

Inquiring about the client's feelings that may affect coping

The nurse is caring for a female client who was recently admitted to the hospital for anorexia nervosa. The nurse enters the client's room and notes that the client is doing vigorous push-ups. Which nursing action is appropriate?

Interrupt the client and offer to take her for a walk.

The nurse is reviewing the record of a client who is hospitalized for treatment of a panic disorder. The nurse notes that the client was admitted by voluntary hospitalization. During the day, the client runs down the hallway and demands release from the hospital. The nurse notes that the client is exhibiting signs/symptoms of anxiety and attempts to assist the client back to the client's hospital room. Which is the next appropriate nursing action at this time?

Notify the registered nurse (RN).

The nurse is assigned to care for a client being admitted to the nursing unit from the emergency department who attempted suicide by ingesting several sleeping pills. The nurse implements which priority action when the client arrives to the unit?

Place the client on one-to-one suicide precautions.

The nurse is caring for a client who has recently developed psychomotor retardation. Based on this information, which behavior would the nurse expect to see in this client?

Slowness of body movements

Of the following actions, which indicate that the relationship between nurse and client may be moving outside professional boundaries?

The client brings the nurse a baked item for their lunch. The nurse is spending more time with the client than the others in the group. The nurse tells a friend that the nurse is the only one who truly understands this client.

The nurse is working with a victim of rape in a clinic setting and assists in developing a plan of care for the client. Which is an inappropriate short-term initial goal?

The client will resolve feelings of fear and anxiety related to the rape trauma.

A mother of a teenage client with an anxiety disorder is concerned about her daughter's progress during discharge. She states that her daughter "stashes food, eats all the wrong things that make her hyperactive," and "hangs out with the wrong crowd." While helping the mother prepare for her daughter's discharge, the nurse should make which suggestion?

The mother should restrict the amount of chocolate and caffeine products in the home.

opioid withdrawal s/s:

withdrawing of CNS depressant, pain blocker - agitation, insomnia - lacrimation, rhinorrhea, sweating, abd cramping, diarrhea (remember how opioids can cause constipation and urinary retention? this is the opposite.) - flu-like s/s, muscle aches Unlike ETOH withdrawal, this won't kill you, although you might wish it would

main concerns c valproate

GI upset (usually temporary) hepatotoxicity, pancreatitis weight gain NOT for pregnancy

The nurse is assigned to care for a client experiencing disturbed thought processes. The nurse is told that the client believes that their food is being poisoned. Which communication technique should the nurse plan to use to encourage the client to eat?

Open-ended questions and silence

During which phase of the nurse-client relationship may the client express ambivalence about the relationship?

Resolution

A client arrives in the emergency department in a crisis state. The client demonstrates signs of profound anxiety and is unable to focus on anything but the object of the crisis and the impact on self. The initial data collection would focus on which information?

The physical condition of the client

A care area is implementing motivational interviewing. What skills will be implemented by the nurse for this technique to be successful? Select all that apply.

self-awareness active listening empathetic linkages strong communication

varenicline is used for?

smoking cessation Chantix Banned for use in commercial truck or bus drivers, pilots, ATC because it can cause unpredictable behavior, mood changes, SI

SSRI onset?

takes a few weeks to get to therapeutic levels

The nurse is meeting with a client experiencing a mood disorder. Which client statement indicates that the nurse-client relationship has been established?

"I feel worthless and have no real use in life.

A client is being encouraged to attend music therapy as part of the individual plan of care. The client refuses to attend and states that he "cannot sing." Which response by the nurse is therapeutic?

"Perhaps you could just enjoy the music without singing."

A client with a potential for violence is exhibiting agitated behavior. The client is using aggressive gestures and making belligerent comments to the other clients and is pacing continually in the hallway. Which comments by the nurse would be therapeutic at this time?

"What is causing you to become agitated?"

A client tells the nurse, "I had to slap my child, I couldn't help that." Which response of the nurse indicates that the nurse is in the state of unknowing?

"What was going on for you when this happened?"

The nurse is performing an admission assessment on a client at high risk for suicide. The nurse should prepare to ask the client which assessment question to elicit data related to this risk? 1. "What are you feeling right now?" 2. "Do you have a plan to commit suicide?" 3. "How many times have you attempted suicide in the past?" 4. "Why were your attempts at suicide unsuccessful in the past?"

2. "Do you have a plan to commit suicide?" Rationale: When assessing for suicide risk, the nurse must determine if the client has a suicide plan. Clients who have a definitive plan pose a greater risk for suicide. Although options 1, 3, and 4 are questions that may provide information that will be helpful in planning care for the client, these questions will not provide information regarding the risk of suicide.

The nurse in the mental health unit is having a conversation with a client diagnosed with posttraumatic stress disorder. The client seems upset and looks anxious. What is the appropriate nursing statement the nurse should make to the client? 1. "Don't worry so much." 2. "I can see that you are upset." 3. "Everything is going to be all right." 4. "Why are you having so much trouble controlling your anxiety?"

2. "I can see that you are upset." Rationale: The correct option is the only one that addresses the client's feelings and concerns. Options 1 and 3 provide false reassurance and place the client's feelings on hold. Option 4 is a nontherapeutic communication technique and will increase the client's anxiety.

A mental health nurse in a psychiatric unit is meeting with a client who has a long history of acting out and violent behavior. The client also is known to have abused drugs on numerous occasions. During the session the client says to the nurse, "I'm feeling much better now, and I'm ready to go straight." Which response by the nurse would be therapeutic? 1. "You have said this many times before!" 2. "Tell me what makes you feel that you are ready." 3. "I have not seen any changes in you to believe that you are ready to go straight." 4. "I'm so glad to hear you talking this way. I will let your health care provider know."

2. "Tell me what makes you feel that you are ready." Rationale: Clients with a long history of acting out and violent behavior and those who have used drugs need to demonstrate motivation to change the behavior, not just verbalization of the behavior. The therapeutic response by the nurse would be directed at assisting the client to look at the behaviors that indicate the change. The correct option is the only one that will provide this direction to the client.

A moderately depressed client who was hospitalized 2 days ago suddenly begins smiling and reporting that the crisis is over. The client says to the nurse, "I'm finally cured." How should the nurse interpret this behavior as a cue to modify the treatment plan? 1. Suggesting a reduction of medication 2. Allowing increased "in-room" activities 3. Increasing the level of suicide precautions 4. Allowing the client off-unit privileges as needed

3. Increasing the level of suicide precautions Rationale: A client who is moderately depressed and has only been in the hospital 2 days is unlikely to have such a dramatic cure. When a depression suddenly lifts, it is likely that the client may have made the decision to harm himself or herself. Suicide precautions are necessary to keep the client safe. The remaining options are therefore incorrect interpretations.

A client comes to the emergency department after an assault and is extremely agitated, trembling, and hyperventilating. What is the priority nursing action for this client? 1. Begin to teach relaxation techniques. 2. Encourage the client to discuss the assault. 3. Remain with the client until the anxiety decreases. 4. Place the client in a quiet room alone to decrease stimulation.

3. Remain with the client until the anxiety decreases. Rationale: This client is in a severe state of anxiety. When a client is in a severe or panic state of anxiety, it is crucial for the nurse to remain with the client. The client in a severe state of anxiety would be unable to learn relaxation techniques. Discussing the assault at this point would increase the client's level of anxiety further. Placing the client in a quiet room alone may also increase the anxiety level.

The nurse in the mental health unit is performing an assessment in a client who has a history of multiple somatic complaints involving several organ systems. Diagnostic studies revealed no organic pathology. The care plan developed for this client will reflect that the client is experiencing which disorder? 1. Depression 2. Schizophrenia 3. Somatization disorder 4. Obsessive-compulsive disorder

3. Somatization disorder Rationale: Somatization disorder is characterized by a long history of multiple physical problems with no satisfactory organic explanation. The clinical findings associated with schizophrenia, depression, and obsessive-compulsive disorder are unrelated to somatic complaints.

What occurs during the working phase of the nurse-client relationship?

Evaluation of mutually identified goals

During the termination phase, a client begins to raise old problems that have already been resolved. Which would be appropriate nursing responses?

Reassure the client that they already covered these issues. Review with the client the learned methods to control the problems.

A woman is brought to the emergency department in a severe state of anxiety after witnessing a devastating car accident that killed two people. Which nursing action should the nurse do first?

Take the client to a quiet room.

A client with a phobia will be treated for the condition using a behavior modification technique known as systematic desensitization. The nurse describes the components of this form of therapy to the client and reinforces which client instruction?

The client will be introduced to short periods of exposure to the phobic object while in a relaxed state.

Which observation should lead the nurse manager to recognize that countertransference is affecting the therapeutic effectiveness of an individual nurse on the unit?

The nurse frequently refers to an elderly, cognitively impaired client as "my granny"

It is brought to the nurse administrator's attention that a nurse has developed an intimate relationship with a client. Which behavior indicates the nurse has engaged in an intimate relationship with a client? The nurse is having dinner with a client outside the hospital premises.

The nurse is having dinner with a client outside the hospital premises.

Teaching for disulfiram?

*avoid all sources of ETOH*, including mouthwashes, aftershave, Nyquil, vanilla extract (which seems a little OTT; as if a teaspoon of vanilla divided between 24 cookies and baked is going to do anything, but whatever: so saith ATI) *will continue to make you feel like hell for a couple weeks* after d/c'ing, so don't think you can skip your dose that day to go party that night, you're going to have to *plan ahead* to fall off the wagon

The nurse is educating a community group about risk factors for suicide and knows a member needs further teaching when which criteria are chosen as risk factors? Select all that apply.

-Age less than 32 years -Practicing a religion -Married over 10 years

The nurse is caring for a client with long-term Alzheimer's disease (AD). Which are some of the behavioral manifestations the nurse should expect to observe? Select all that apply.

-Apraxia -Aphasia -Agnosia -Hyperorality

The student nurse is learning about leadership and management. The student knows that which are the main styles of group leadership? Select all that apply.

-Autocratic Leader -Democratic Leader -Laissez-Faire Leader

The nursing instructor is helping students learn about bioethics, which is the study of specific ethical questions that arise in health care. The instructor reviews with the students which basic principles of bioethics? Select all that apply.

-Autonomy -Beneficence -Veracity -Fidelity -Justice

The nurse is assessing a client diagnosed with posttraumatic stress disorder (PTSD). The nurse knows that according to current references, PTSD signs/symptoms can be grouped into which three main categories? Select all that apply.

-Avoidance -Hyperarousal -Reexperiencing

The nurse is admitting a client with a diagnosis of agoraphobia. Which behaviors exhibited by the client would support this diagnosis? Select all that apply.

-Being on a bridge -Riding in an elevator -Being alone at home -Traveling in an airplane

The licensed practical nurse is assisting in the admittance of a client who has been involuntarily committed to the behavioral health unit. Which actions by the client before hospitalization led to the commitment? Select all that apply.

-Client threatened to commit suicide -Client threatened to kidnap his spouse

Which nursing interventions are appropriate for a hospitalized client with mania who is exhibiting manipulative behavior? Select all that apply.

-Communicate expected -Follow through -Assist the client -Be clear w/ the client

A client on the mental health unit is exhibiting distancing and does not speak to his/her family or visitors. Which are some other adverse relationship patterns? Select all that apply.

-Cutoffs -Conflict -Over involvement

The nurse is assessing a client who has been diagnosed with Alzheimer's disease. The nurse knows that in the initial stages the client and family try to hide deficits in memory. Which are some of the defense mechanisms related to the progression of the disease? Select all that apply.

-Denial -Confabulation -Perseveration -Avoidance of questions

A client is admitted to the mental health unit with a diagnosis of possible somatic symptom disorder. Besides anxiety, the nursing assessment is especially important in identifying which client signs/symptoms are contributing to the somatic symptom disorder? Select all that apply.

-Depression -Substance Abuse -Adverse Childhood Events -PTSD

The nurse is assessing a client who takes antipsychotic medication for which signs/symptoms that might indicate the development of neuroleptic malignant syndrome? Select all that apply.

-Diaphoretic -Temperature of 104.8 -Blood pressure of 210/130 mmHg

The nurse is caring for a client with depression in the mental health unit who is refusing to take the prescribed oral antidepressant. Which are the nurse's best actions in response to this client's medication refusal? Select all that apply.

-Document the refusal of medication -Notify the RN -Ask the client why he is refusing the medication

The nurse is reading about the four different levels of anxiety. Which different categories distinguish and describe each level? Select all that apply.

-Effects on problem solving -Effects on perceptual field -Physical and other defining characteristics

The licensed practical nurse is assisting the registered nurse in admitting a client with an exacerbation of schizophrenia and knows that which signs/symptoms displayed by the client are considered positive symptoms? Select all that apply.

-Hallucinations -Delusions -Neologisms

The nurse is admitting a victim abuse client to the mental health unit with a diagnosis of severe anxiety. The nurse notes which signs/symptoms that indicate it is difficult for the victim to talk about the situation? Select all that apply.

-Hesitation -Lack of eye contact -Using vague statements such as,...

The nurse is having a therapeutic discussion with a client and knows that which statements by the client should be immediately reported to the charge nurse? Select all that apply.

-I hid my silverware from dinner last night -I know that by this time tomorrow all my troubles will be over

The nurse is caring for a client who is hospitalized because of severe depression. Which statements would be most helpful in assisting this client? Select all that apply.

-I notice you are wearing a blue shirt. -Do you have any plans of harming yourself? -I will sit here with you even if you choose not to talk with me.

While discharge planning for a female teenager with anorexia nervosa, the nurse suggests that the teenager attends a meeting of the local chapter of the National Association of Anorexia Nervosa and Associated Disorders. Which responses by the teenager indicate that she will likely be compliant with this suggestion? Select all that apply.

-I'm going to do whatever... -I'll go and participate...

The nurse in the mental health clinic hears a client yelling and threatening to hurt his sister. The nurse reports this episode to the mental health therapist. Which should the nurse anticipate the therapist to do? Select all that apply.

-Identify the specific person being threatened -Take appropriate action to protect the identified victim -Assess and predict the client's danger of violence toward another

A client is admitted to a psychiatric unit for observation following severe anxiety attacks. On admission, the client states, "There's nothing wrong with me. I shouldn't even be here. I am taking up a room, and there is probably someone else who really needs it." Although the nurse interprets this response as denial, which findings support a severe level of anxiety? Select all that apply.

-Inability to think clearly -Inability to problem solve

The nurse is assessing a client with bipolar disorder who is taking lithium carbonate and who has a lithium level of 1.7 mEq/L. The nurse would expect to find which sign/symptoms of lithium toxicity associated with this level? Select all that apply.

-Incoordination -Mental confusion -Muscle hyperirritability

An oriented client is scheduled to have aversion therapy to change behavior. Before initiating any aversive protocol, the therapist, treatment team, or society must answer which questions? Select all that apply.

-Is it in the best interest of society? -Does its use violate the client's rights? -Is this therapy in the best interest of the client?

The nurse awakens a client on the inpatient psychiatric unit for breakfast. The client replies, "Do you realize it's Sunday? I've worked hard here all week and this is my day of rest. I'll get up at 11:30." Which would be the nurse's best response?

-Let me know if you change your mind...

Which data indicate to the nurse that a client is experiencing effective coping following the loss of a spouse? Select all that apply.

-Looks at old snapshot of family -Visits the spouse's grave once a month -Visits the senior citizens' center once...

An adolescent client is admitted to the inpatient unit after medical stabilization for an overdose of acetaminophen. The history identifies that her boyfriend broke up with her 2 weeks ago and that she hasn't been eating well, resulting in a loss of 15 pounds. The nurse assists in developing a plan of care that includes which interventions? Select all that apply.

-Making -Providing -Ensuring

Which are the major roles the nurse can play in advocating for psychiatric evaluation and intervention for clients with a history of depression, schizophrenia, obsessive-compulsive disorder, generalized anxiety disorder, or bipolar disorder? Select all that apply.

-Medication management -Monitoring and Documenting Behavioral -Notifying the health care provider... -Planning care for the needs of those...

The nurse is caring for a client who is diagnosed with anxiety. The nurse knows that according to Hildegard Peplau, there are different levels of anxiety that include which? Select all that apply.

-Mild -Panic -Severe -Moderate

The nurse caring for a client who has been diagnosed with stage 3 Alzheimer's disease should expect to observe which behaviors in this client? Select all that apply.

-Misplacing a valuable object -Difficulty coming up with the right word

A client diagnosed with schizophrenia is experiencing an acute dystonic reaction. Which interventions should the licensed practical nurse (LPN) initiate? Select all that apply.

-Monitor airway -Notify the RN -Remain with the client to provide support -Administer the prescribed intramuscular antiparkinsonian medication

Which are appropriate interventions for caring for the client undergoing alcohol withdrawal? Select all that apply.

-Monitor vital signs -Provide a safe environment -Address hallucinations -Provide reality orientation

The nurse is assisting in a group therapy session. Besides cost savings, which advantages does group therapy have over individual therapy? Select all that apply.

-Mutual Learning -Increased Feedback -Instilling a sense of belonging -An opportunity to practice new skills in a relative environment

The nurse is assisting in a group therapy session. Besides cost savings, which advantages does group therapy have over individual therapy? Select all that apply.

-Mutual Learning -Increased Feedback -Instilling a sense of belonging -An opportunity to practice...

A client is being seen at the primary care clinic for her annual gynecological examination. Which client statements are most likely associated with potential intimate partner abuse? Select all that apply.

-My husband always brings... -I have bruises all over my... -My boyfriend yells and ...

The nurse is assessing a client diagnosed with severe anxiety. Which objective data should the nurse expect to find? Select all that apply.

-Oblivious to surroundings -Unable to focus on anything -Engaging in purposeless activities -Showing unproductive relief behavior

The nurse is assessing a client with a diagnosis of bipolar affective disorder-mania. Which characteristics appropriately describe this client's diagnosis? Select all that apply.

-Outlandish behaviors -Purposeless arousal and movement -Grandiose delusions of being King Arthur -Incessant talking that includes sexual innuendos

The nurse is collecting data on a newly admitted client with conversion disorder. The nurse knows which voluntary motor or sensory function deficits might be present in this client? Select all that apply.

-Paralysis -Blindness -Paresthesia -Movement Disorder

A client who has successfully adjusted to a colostomy declines the invitation to speak to a support group on the subject of alteration in body image. The client reports an extreme fear of public speaking. The nurse recognizes that this client is suffering from social phobia. Which are some other manifestations of social phobias? Select all that apply.

-Performing badly on stage -Looking awkward while eating or drinking in public -Not being able to answer questions in a classroom -Fear of saying something that sounds foolish in public

Milieu therapy is prescribed for a client on the psychiatric unit. The nurse knows that besides overcrowding on the unit, milieu characteristics conducive to violence include which factors? Select all that apply.

-Poor limited setting -Staff inexperience -Provocative or controlling staff -Arbitrary revocation of privileges

The psychiatric nurse knows that a therapeutic nurse-client relationship includes which specific goals and functions? Select all that apply.

-Promoting self-care and independence -Facilitating communication of distressing thoughts and feelings -Helping clients examine self-defeating behaviors and test alternatives -Assisting clients with problem solving to help facilitate activities of daily living

The nurse collecting data from a 35-year-old client determines that the client has gained more than 100 pounds in an 18-month period. The client confided in the nurse that she was sexually molested at the age of 7 and began putting on weight after that time. The client presently weighs 422 pounds. The nurse determines that obesity for this client most likely represents which reason?

-Protection from the risk of intimacy

The nurse on the mental health unit is caring for a client with a history of alcoholism. Aversion conditioning has been chosen as the treatment for this client because other less drastic measures have failed to produce the desired effects. Which are some paradigms or clear examples of aversion conditioning? Select all that apply.

-Punishment -Avoidance Training -Pairing of a maladaptive behavior

The nurse is caring for a client who was recently admitted to the inpatient unit of a psychiatric hospital with a diagnosis of delusions. Which are some therapeutic communication interventions the nurse needs to use when communicating with this client? Select all that apply.

-Refer to hallucinations as if they are real -Ask the client directly about the hallucinations -Watch the client for cues... -Address any underlying emotion, need, or them...

The nurse is caring for a client with a somatic disorder and knows that which interventions would be most helpful to this client? Select all that apply.

-Reinforce the client's problem-solving abilities -Assess "secondary gains" that the somatic illness provides the client.

The nurse is caring for a client who has been diagnosed with a dissociative disorder. Which interventions should the nurse use in providing care for the client? Select all that apply.

-Request that the client perform undemanding, self-care tasks -Reinforce teaching the client techniques to maintain present reality -Assist the client to reestablish relationships w/ significant others

The student nurse is studying the cellular composition of the brain composed of approximately 100 billion neurons or nerve cells. Although neurons come in a great variety of shapes and sizes, all carry out the same three types of physiological actions. Which are these types of actions? Select all that apply.

-Respond to stimuli -Conduct electrical impulses -Release chemicals called neurotransmitters

The nurse in the mental health unit reviews the therapeutic and nontherapeutic communication techniques with a nursing student. Which are therapeutic communication techniques? Select all that apply.

-Restating -Listening -Maintaining neutral responses -Providing acknowledgement and feedback

The nurse on the mental health unit is collecting data on a client diagnosed with obsessive-compulsive disorder (OCD). The nurse expects to note which behavioral characteristics of OCD? Select all that apply.

-Rigidity -Inflexibility -Repetitive thoughts -Ritualistic behavior

The nurse prepares the plan of care for a client with late-stage Alzheimer's disease who resides in a long-term care facility. Which would be priority concerns to include? Select all that apply.

-Risk for injury -Risk for infection -Risk for aspiration -Impaired verbal communication

The nurse is admitting a client who has a history of bipolar disorder to the hospital, and the primary health care provider has indicated that the client is currently in the manic phase. Which actions should the nurse include in the plan of care? Select all that apply.

-Set limits on behavior -Distract or redirect the client -Decrease environmental stimulation -Provide high caloric nutritional intake

A client with a potential for violence is exhibiting agitated behavior. The client is using aggressive gestures and making belligerent comments to the other clients and is pacing continually in the hallway. The nurse is considering seclusion and restraints for this client even though staffing is lacking for close supervision and direct observation. Which are some contraindications to seclusion and restraints without close supervision and observation? Select all that apply.

-Severe -Extremeley -Desire for -Delirium -Severe drug reactions...

Which nursing interventions are most helpful when caring for a client who is displaying signs/symptoms of panic level anxiety? Select all that apply.

-Speak slowly -Use simple statements -Provide the client with high-calorie beverages

The registered nurse has written an outcome statement of, "Client will feel less anxious by the end of session," for a client with generalized anxiety disorder. Which interventions should the licensed practical nurse use to assist this client in meeting this goal? Select all that apply.

-Stay with the client -Administer anxiolytics medications -Ensure the client is in an environment...

The nurse is working with an older client who has a diagnosis of depression. To work most effectively with this client, the nurse recalls that which information is accurate regarding depression and the older client? Select all that apply.

-Suicide is a frequent cause of death among the older population. -Some indications of dementia may actually originate as depression. -Depression in an older person is likely to have physical manifestations.

The nurse is gathering data from a client diagnosed with a phobia. Which are some of the clinically recognized names of common phobias? Select all that apply.

-Zoophobia -Xenophobia -Agoraphobia -Glossophobia

Which are characteristics of the termination stage of group development? Select all that apply. 1. The group evaluates the experience. 2. The real work of the group is accomplished. 3. Group interaction involves superficial conversation. 4. Group members become acquainted with each other. 5. Some structuring of group norms, roles, and responsibilities takes place. 6. The group explores members' feelings about the group and the impending separation.

1 and 6 Rationale: The stages of group development include the initial stage, the working stage, and the termination stage. During the initial stage, the group members become acquainted with each other, and some structuring of group norms, roles, and responsibilities takes place. During the initial stage, group interaction involves superficial conversation. During the working stage, the real work of the group is accomplished. During the termination stage, the group evaluates the experience and explores members' feelings about the group and the impending separation.

The nurse is preparing a client for electroconvulsive therapy (ECT). The family of the client asks the nurse about this treatment. The nurse responds, knowing that which statements are accurate regarding this treatment? Select all that apply. 1. The average series involves 6 to 12 treatments. 2. Some confusion may be noted after the procedure. 3. Memory loss will occur but will resolve with time. 4. This treatment is a permanent cure to the condition. 5. This treatment is tried before the use of medications.

1, 2, 3 Rationale: ECT as a form of treatment is considered when medication therapy has failed, the client is at high risk for suicide, or depression is judged to be overwhelmingly severe. Treatments are administered three times a week, with an average series involving 8 to 12 treatments over a duration of 2 to 4 weeks. The most common side effect is amnesia for events occurring near the period of treatment. Memory deficits may occur and tend to resolve with time. This treatment is not a permanent cure to the client's condition.

The nurse is developing a plan of care for a client with depression whose food intake is poor. The nurse should include which interventions in the plan of care? Select all that apply. 1. Assist the client in selecting foods from the food menu. 2. Offer high-calorie fluids throughout the day and evening. 3. Allow the client to eat alone in the room if the client requests to do so. 4. Offer small high-calorie, high-protein snacks during the day and evening. 5. Select the foods for the client to be sure that the client eats a balanced diet.

1, 2, 4 Rationale: In caring for a client with depression whose nutritional intake is poor, the nurse should remain with the client during the meal. The nurse also should assist the client in selecting foods from the menu because the client is more likely to eat the foods that he or she likes. Offering small high-calorie, high-protein snacks and high-calorie fluids throughout the day and evening are appropriate interventions for the client to maintain nutrition.

The nurse in the mental health unit recognizes which as being therapeutic communication techniques? Select all that apply. 1. Restating 2. Listening 3. Asking the client, "Why?" 4. Maintaining neutral responses 5. Providing acknowledgment and feedback 6. Giving advice and approval or disapproval

1, 2, 4, 5 Rationale: Therapeutic communication techniques include listening, maintaining silence, maintaining neutral responses, using broad openings and open-ended questions, focusing and refocusing, restating, clarifying and validating, sharing perceptions, reflecting, providing acknowledgment and feedback, giving information, presenting reality, encouraging formulation of a plan of action, providing nonverbal encouragement, and summarizing. Asking why is often interpreted as being accusatory by the client and should also be avoided. Providing advice or giving approval or disapproval are barriers to communication.

Which nursing interventions are appropriate for a hospitalized client with mania who is exhibiting manipulative behavior? Select all that apply. 1. Communicate expected behaviors to the client. 2. Ensure that the client knows that they are not in charge of the nursing unit. 3. Assist the client in identifying ways of setting limits on personal behaviors. 4. Follow through about the consequences of behavior in a nonpunitive manner. 5. Enforce rules by informing the client that they will not be allowed to attend therapy groups. 6. Have the client state the consequences for behaving in ways that are viewed as unacceptable.

1, 3, 4, 6 Rationale: Interventions for dealing with the client exhibiting manipulative behavior include setting clear, consistent, and enforceable limits on manipulative behaviors; being clear with the client regarding the consequences of exceeding limits set; following through with the consequences in a nonpunitive manner; and assisting the client in identifying means of setting limits on personal behaviors. Ensuring that the client knows that he or she is not in charge of the nursing unit is inappropriate; power struggles need to be avoided. Enforcing rules and informing the client that he or she will not be allowed to attend therapy groups is a violation of a client's rights.

Which statement made by an unlicensed assistive personnel (UAP) indicates to the registered nurse that the UAP understands the concepts related to suicide? 1. "Discussing suicide with a client is not harmful." 2. "Those clients who talk about suicide never do it." 3. "Depressed clients are the only persons who commit suicide." 4. "When a person talks about making suicide threats, the only thing the person wants is attention from family and friends."

1. "Discussing suicide with a client is not harmful." Rationale: An open discussion of suicide will not encourage a client to make a decision to commit suicide and in fact often will help to prevent it. Such a discussion offers the health care professional the opportunity to assess the reality of suicide for the client and take necessary precautions to keep the client safe. Options 2, 3, and 4 are inaccurate statements regarding suicide.

The nurse determines that the wife of an alcoholic client is benefiting from attending an Al-Anon group if the nurse hears the wife make which statement? 1. "I no longer feel that I deserve the beatings my husband inflicts on me." 2. "My attendance at the meetings has helped me to see that I provoke my husband's violence." 3. "I enjoy attending the meetings because they get me out of the house and away from my husband." 4. "I can tolerate my husband's destructive behaviors now that I know they are common with alcoholics."

1. "I no longer feel that I deserve the beatings my husband inflicts on me." Rationale: Al-Anon support groups are a protected, supportive opportunity for spouses and significant others to learn what to expect and to obtain excellent pointers about successful behavioral changes. The correct option is the healthiest response because it exemplifies an understanding that the alcoholic partner is responsible for his behavior and cannot be allowed to blame family members for loss of control. Option 2 is incorrect because the nonalcoholic partner should not feel responsible when the spouse loses control. Option 3 indicates that the group is viewed as an escape, not as a place to work on issues. Option 4 indicates that the wife remains codependent.

A client is planning to attend Overeaters Anonymous. Which statement by the client indicates a need for additional information regarding this self-help group? 1. "The leader is a nurse or psychiatrist." 2. "The members provide support to each other." 3. "People who have a similar problem are able to help others." 4. "It is designed to serve people who have a common problem."

1. "The leader is a nurse or psychiatrist." Rationale: The sponsor of a self-help group is an experienced member of the group. The nurse or psychiatrist may be asked by the group to serve as a resource, but would not be the leader of the group. The remaining options are characteristics of a self-help group.

A client with depression is scheduled to receive three sessions of electroconvulsive therapy (ECT). The client asks the nurse about the length of time it will take for improvement in the condition. The nurse should tell the client he or she will see improvement approximately how long after the three treatments? 1. 1 week 2. 3 weeks 3. 4 weeks 4. 8 weeks

1. 1 week Rationale: Health care providers generally administer ECT treatments three times a week, with an average series including 8 to 12 treatments. After three sessions of ECT, the client should start to demonstrate improvement in 1 week. Options 2, 3, and 4 are incorrect.

Which client is most at risk for committing suicide? 1. A 75-year-old client with metastatic cancer 2. A 71-year-old client with a cardiac disorder 3. A 24-year-old client who just had an argument with her roommate 4. A 30-year-old newly divorced client who states she has custody of the children

1. A 75-year-old client with metastatic cancer Rationale: The person most at risk for suicide is the client with terminal illness. Other high-risk groups include adolescents, drug abusers, persons who have experienced recent losses, those who have few or no social supports, and those with a history of suicide attempts and a suicide plan.

A client is preparing to attend a Gamblers Anonymous meeting for the first time. The nurse should tell the client that which is the first step in this 12-step program? 1. Admitting to having a problem 2. Substituting other activities for gambling 3. Stating that the gambling will be stopped 4. Discontinuing relationships with people who gamble

1. Admitting to having a problem Rationale: The first step in the 12-step program is to admit that a problem exists. Substituting other activities for gambling may be a strategy but it is not the first step. The remaining options are not realistic strategies for the initial step in a 12-step program.

A client with diabetes mellitus is told that amputation of the leg is necessary to sustain life. The client is very upset and tells the nurse, "This is all my health care provider's fault. I have done everything I've been asked to do!" Which nursing interpretation is best for this situation? 1. An expected coping mechanism 2. An ineffective coping mechanism 3. A need to notify the hospital lawyer 4. An expression of guilt on the part of the client

1. An expected coping mechanism Rationale: The nurse needs to be aware of the effective and ineffective coping mechanisms that can occur in a client when loss is anticipated. The expression of anger is known to be a normal response to impending loss, and the anger may be directed toward the self, God or other spiritual being, or caregivers. Notifying the hospital lawyer is inappropriate. Guilt may or may not be a component of the client's feelings, and the data in the question do not indicate that guilt is present.

The home health nurse visits a client at home and determines that the client is dependent on drugs. During the assessment, which action should the nurse take to plan appropriate nursing care? 1. Ask the client why he started taking illegal drugs. 2. Ask the client about the amount of drug use and its effect. 3. Ask the client how long he thought that he could take drugs without someone finding out. 4. Not ask any questions for fear that the client is in denial and will throw the nurse out of the home.

1. Ask the client why he started taking illegal drugs. Rationale: Whenever the nurse carries out an assessment for a client who is dependent on drugs, it is best for the nurse to attempt to elicit information by being nonjudgmental and direct. Option 1 is incorrect because it is judgmental and off-focus, and reflects the nurse's bias. Option 3 is incorrect because it is judgmental, insensitive, and aggressive, which is nontherapeutic. Option 4 is incorrect because it indicates passivity on the nurse's part and uses rationalization to avoid the therapeutic nursing intervention.

A hospitalized client with a history of alcohol abuse tells the nurse, "I am leaving now. I have to go. I don't want any more treatment. I have things that I have to do right away." The client has not been discharged and is scheduled for an important diagnostic test to be performed in 1 hour. After the nurse discusses the client's concerns with the client, the client dresses and begins to walk out of the hospital room. What action should the nurse take? 1. Call the nursing supervisor. 2. Call security to block all exit areas. 3. Restrain the client until the health care provider (HCP) can be reached. 4. Tell the client that the client cannot return to this hospital again if the client leaves now.

1. Call the nursing supervisor. Rationale: Most health care facilities have documents that the client is asked to sign relating to the client's responsibilities when the client leaves against medical advice. The client should be asked to wait to speak to the HCP before leaving and to sign the "against medical advice" document before leaving. If the client refuses to do so, the nurse cannot hold the client against the client's will. Therefore, in this situation, the nurse should call the nursing supervisor. The nurse can be charged with false imprisonment if a client is made to believe wrongfully that he or she cannot leave the hospital. Restraining the client and calling security to block exits constitutes false imprisonment. All clients have a right to health care and cannot be told otherwise.

A home care nurse making an initial home visit notes that a client is taking donepezil hydrochloride (Aricept). The nurse questions the client's spouse about a history of which disorder that is treated with this medication? 1. Dementia 2. Schizophrenia 3. Seizure disorder 4. Obsessive-compulsive disorder

1. Dementia Rationale: Donepezil hydrochloride is a cholinergic agent that is used in the treatment of mild to moderate dementia of the Alzheimer's type. It increases concentration of acetylcholine, which slows the progression of Alzheimer's disease. The other options are incorrect and are not indications for use of this medication.

A client admitted to a mental health unit for treatment of psychotic behavior spends hours at the locked exit door shouting, "Let me out. There's nothing wrong with me. I don't belong here." What defense mechanism is the client implementing? 1. Denial 2. Projection 3. Regression 4. Rationalization

1. Denial Rationale: Denial is refusal to admit to a painful reality, which is treated as if it does not exist. In projection, a person unconsciously rejects emotionally unacceptable features and attributes them to other persons, objects, or situations. Regression allows the client to return to an earlier, more comforting, although less mature, way of behaving. Rationalization is justifying illogical or unreasonable ideas, actions, or feelings by developing acceptable explanations that satisfy the teller and the listener.

The emergency department nurse is caring for an adult client who is a victim of family violence. Which priority instruction should be included in the discharge instructions? 1. Information regarding shelters 2. Instructions regarding calling the police 3. Instructions regarding self-defense classes 4. Explaining the importance of leaving the violent situation

1. Information regarding shelters Rationale: Tertiary prevention of family violence includes assisting the victim after the abuse has already occurred. The nurse should provide the client with information regarding where to obtain help, including a specific plan for removing the self from the abuser and information regarding escape, hotlines, and the location of shelters. An abused person is usually reluctant to call the police. Teaching the victim to fight back is not the appropriate action for the victim when dealing with a violent person. Explaining the importance of leaving the violent situation is important, but a specific plan is necessary.

The nurse has been observing a client closely who has been displaying aggressive behaviors. The nurse observes that the behavior displayed by the client is escalating. Which nursing intervention is least helpful to this client at this time? 1. Initiate confinement measures. 2. Acknowledge the client's behavior. 3. Assist the client to an area that is quiet. 4. Maintain a safe distance from the client.

1. Initiate confinement measures. Rationale: During the escalation period, the client's behavior is moving toward loss of control. Nursing actions include taking control, maintaining a safe distance, acknowledging behavior, moving the client to a quiet area, and medicating the client if appropriate. To initiate confinement measures during this period is inappropriate. Initiation of confinement measures, if needed, is most appropriate during the crisis period.

The nurse understands that which best describes Gestalt therapy? 1. It emphasizes self-expression, self-exploration, and self-awareness in the present. 2. It promotes the individual's comfort in the group, which then transfers to other relationships. 3. The therapist focuses on how irrational beliefs and thoughts contribute to psychological distress. 4. The therapist's goal is to help others express their feelings toward one another during group sessions.

1. It emphasizes self-expression, self-exploration, and self-awareness in the present. Rationale: Gestalt therapy emphasizes self-expression, self-exploration, and self-awareness in the present. The client and therapist focus on everyday problems and try to solve them. Interpersonal group therapy promotes the individual's comfort in the group, which then transfers to other relationships. In rational emotive therapy, the therapist focuses on how irrational beliefs and thoughts contribute to psychological distress. In Rogerian therapy, the therapist's goal is to help others express their feelings toward one another during group sessions.

Which type of therapeutic approach has the characteristic that all team members are seen as equally important in helping clients meet their goals? 1. Milieu therapy 2. Interpersonal therapy 3. Behavior modification 4. Rational emotive therapy

1. Milieu therapy Rationale: All treatment team members are viewed as significant and valuable to the client's successful treatment outcomes in milieu therapy. Interpersonal therapy is based on a one-to-one or group therapy approach in which the therapist-client relationship is often used as a way for the client to examine other relationships in his or her life. Behavior modification is based on rewards and punishment. Rational emotive therapy deals with the correction of distorted thinking.

When reviewing the admission assessment, the nurse notes that a client was admitted to the mental health unit involuntarily. Based on this type of admission, the nurse should provide which intervention for this client? 1. Monitor closely for harm to self or others. 2. Assist in completing an application for admission. 3. Supply the client with written information about their mental illness. 4. Provide an opportunity for the family to discuss why they felt the admission was needed.

1. Monitor closely for harm to self or others. Rationale: Involuntary admission is necessary when a person is a danger to self or others or is in need of psychiatric treatment regardless of the client's willingness to consent to the hospitalization. A written request is a component of a voluntary admission. Providing written information regarding the illness is likely premature initially. The family may have had no role to play in the client's admission.

The nurse is caring for a client who is at risk for suicide. What is the priority nursing action for this client? 1. Provide authority, action, and participation. 2. Display an attitude of detachment, confrontation, and efficiency. 3. Demonstrate confidence in the client's ability to deal with stressors. 4. Provide hope and reassurance that the problems will resolve themselves.

1. Provide authority, action, and participation. Rationale: A crisis is an acute, time-limited state of disequilibrium resulting from situational, developmental, or societal sources of stress. A person in this state is temporarily unable to cope with or adapt to the stressor by using previous coping mechanisms. The person who intervenes in this situation (the nurse) "takes over" for the client (authority) who is not in control and devises a plan (action) to secure and maintain the client's safety. When this has occurred, the nurse works collaboratively with the client (participates) in developing new coping and problem-solving strategies.

The nurse observes that a client is pacing, agitated, and presenting aggressive gestures. The client's speech pattern is rapid, and affect is belligerent. Based on these observations, what is the nurse's immediate priority of care? 1. Provide safety for the client and other clients on the unit. 2. Provide the clients on the unit with a sense of comfort and safety. 3. Assist the staff in caring for the client in a controlled environment. 4. Offer the client a less stimulating area to calm down in and gain control.

1. Provide safety for the client and other clients on the unit. Rationale: Safety of the client and other clients is the priority. The correct option is the only one that addresses the safety needs of the client as well as those of the other clients.

The nurse is conducting a group therapy session. During the session, a client diagnosed with mania consistently disrupts the group's interactions. Which intervention should the nurse initially implement? 1. Setting limits on the client's behavior 2. Asking the client to leave the group session 3. Asking another nurse to escort the client out of the group session 4. Telling the client that they will not be able to attend any future group sessions

1. Setting limits on the client's behavior Rationale: Manic clients may be talkative and can dominate group meetings or therapy sessions by their excessive talking. If this occurs, the nurse initially would set limits on the client's behavior. Initially, asking the client to leave the session or asking another person to escort the client out of the session is inappropriate. This may agitate the client and escalate the client's behavior further. Barring the client from group sessions is also an inappropriate action because it violates the client's right to receive treatment and is a threatening action.

Which behavior observed by the nurse indicates a suspicion that a depressed adolescent client may be suicidal? 1. The adolescent gives away a DVD and a cherished autographed picture of a performer. 2. The adolescent runs out of the therapy group, swearing at the group leader, and runs to her room. 3. The adolescent becomes angry while speaking on the telephone and slams down the receiver. 4. The adolescent gets angry with her roommate when the roommate borrows the client's clothes without asking.

1. The adolescent gives away a DVD and a cherished autographed picture of a performer. Rationale: A depressed suicidal client often gives away that which is of value as a way of saying goodbye and wanting to be remembered. Options 2, 3, and 4 deal with anger and acting-out behaviors that are often typical of any adolescent.

A mental health nurse notes that a client with schizophrenia is exhibiting an immobile facial expression and a blank look. Which should the nurse document in the client's record? 1. The client has a flat affect. 2. The client has an inappropriate affect. 3. The client is exhibiting bizarre behavior. 4. The client's emotional responses exhibit a blunted affect.

1. The client has a flat affect. Rationale: A flat affect is manifested as an immobile facial expression or blank look. An inappropriate affect refers to an emotional response to a situation that is incongruent with the tone of the situation. A bizarre affect such as grimacing, laughing, and self-directed mumbling is marked when the client is unable to relate logically to the environment. A blunted affect is a minimal emotional response or outward affect that typically does not coincide with the client's inner emotions.

The nurse is performing an assessment on a client with dementia. Which data gathered during the assessment indicates a manifestation associated with dementia? 1. Uses confabulation 2. Improvement in sleeping 3. Absence of sundown syndrome 4. Presence of personal hygienic care

1. Uses confabulation Rationale: The clinical picture of dementia ranges from mild cognitive deficits to severe, life-threatening alterations in neurological functioning. For the client to use confabulation or the fabrication of events or experiences to fill in memory gaps is not unusual. Often, lack of inhibitions on the part of the client may constitute the first indication of something being "wrong" to the client's significant others (e.g., the client may undress in front of others, or the formerly well-mannered client may exhibit slovenly table manners). As the dementia progresses, the client will have difficulty sleeping and episodes of wandering or sundowning.

A client experiencing disturbed thought processes believes that his food is being poisoned. Which communication technique should the nurse use to encourage the client to eat? 1. Using open-ended questions and silence 2. Sharing personal preference regarding food choices 3. Documenting reasons why the client does not want to eat 4. Offering opinions about the necessity of adequate nutrition

1. Using open-ended questions and silence Rationale: Open-ended questions and silence are strategies used to encourage clients to discuss their problems. Sharing personal food preferences is not a client-centered intervention. The remaining options are not helpful to the client because they do not encourage the client to express feelings. The nurse should not offer opinions and should encourage the client to identify the reasons for the behavior.

The nurse is developing a plan of care for the client with a diagnosis of paranoia and should include which interventions in the plan of care? Select all that apply. 1. Provide a warm approach to the client. 2. Ask permission before touching the client. 3. Eliminate physical contact with the client. 4. Defuse any anger or verbal attacks with a nondefensive stance. 5. Use simple and clear language when communicating with the client.

2, 3, 4, 5 Rationale: When caring for a client with paranoia, the nurse should ask permission if touch is necessary because touch may be interpreted as a sexual or physical assault. The nurse must eliminate any physical contact and not touch the client. The anger that a paranoid client expresses often is displaced, and when a staff member becomes defensive, both client and staff anger may escalate. Simple and clear language should be used in speaking to the client to prevent misinterpretation and to clarify the nurse's intent and action. The nurse should avoid a warm approach because warmth can be frightening to a person who needs emotional distance.

A client is admitted with a recent history of severe anxiety following a home invasion and robbery. During the initial assessment interview, which statement by the client would indicate to the nurse the possible diagnosis of posttraumatic stress disorder? Select all that apply. 1. "I'm afraid of spiders." 2. "I keep reliving the robbery." 3. "I see his face everywhere I go." 4. "I don't want anything to eat now." 5. "I might have died over a few dollars in my pocket." 6. "I have to wash my hands over and over again many times."

2, 3, 5 Rationale: Reliving an event, experiencing emotional numbness (facing possible death), and having flashbacks of the event (seeing the same face everywhere) are all common occurrences with posttraumatic stress disorder. The statement. "I'm afraid of spiders," is more relative to having a phobia. The statement "I have to wash my hands over and over again many times" describes ritual compulsive behaviors to decrease anxiety for someone with obsessive compulsive disorder. Stating "I don't want anything to eat now" is vague and could relate to numerous conditions.

A nursing student is assisting with the care of a client with a chronic mental illness. The nurse informs the student that a behavior modification approach (operant conditioning) will be used in treatment for the client. Which statement by the student indicates a need for further information about the therapy? 1. "It uses positive reinforcement." 2. "It uses negative reinforcement." 3. "It increases social behaviors in the client." 4. "It increases the level of self-care in the client."

2. "It uses negative reinforcement." Rationale: Operant conditioning entails rewarding a client for desired behaviors and is the basis for behavior modification. It uses a positive reinforcement approach. Options 1, 3, and 4 are accurate characteristics of this form of therapy.

The mental health nurse is conducting a group therapy session and is monitoring a client with a diagnosis of agoraphobia who has been attending the sessions for several months. The nurse notes that the client is cooperative, sharing with peers, and making appropriate suggestions during group discussions. How should the nurse interpret this behavior? 1. Manipulation 2. Improvement 3. Attention seeking 4. Desire to be accepted

2. Improvement Rationale: The behaviors identified in the question indicate improvement in the client's condition. The question presents no information indicating that the client is being manipulative. Acting out is attention-seeking behavior. All clients have a desire to be accepted.

The nurse is caring for a female client who was admitted to the mental health unit recently for anorexia nervosa. The nurse enters the client's room and notes that the client is engaged in rigorous push-ups. Which nursing action is most appropriate? 1. Interrupt the client and weigh her immediately. 2. Interrupt the client and offer to take her for a walk. 3. Allow the client to complete her exercise program. 4. Tell the client that she is not allowed to exercise rigorously.

2. Interrupt the client and offer to take her for a walk. Rationale: Clients with anorexia nervosa frequently are preoccupied with rigorous exercise and push themselves beyond normal limits to work off caloric intake. The nurse must provide for appropriate exercise and place limits on rigorous activities. The correct option stops the harmful behavior yet provides the client with an activity to decrease anxiety that is not harmful. Weighing the client immediately reinforces the client's preoccupation with weight. Allowing the client to complete the exercise program can be harmful to the client. Telling the client that she is not allowed to complete the exercise program will increase the client's anxiety.

The nurse in the emergency department is caring for a young female victim of sexual assault. The client's physical assessment is complete, and physical evidence has been collected. The nurse notes that the client is withdrawn, confused, and at times physically immobile. How should the nurse interpret these behaviors? 1. Signs of depression 2. Normal reactions to a devastating event 3. Evidence that the client is a high suicide risk 4. Indicative of the need for hospital admission

2. Normal reactions to a devastating event Rationale: During the acute phase of the rape crisis, the client can display a wide range of emotional and somatic responses. The symptoms noted indicate a normal reaction. Options 1, 3, and 4 are incorrect interpretations.

A client has been diagnosed with major depression. The nurse notes that the client is not eating adequately and at times refuses to eat. What should the nurse plan to do to meet the client's nutritional needs? 1. Force foods and fluids. 2. Provide small, frequent meals. 3. Provide snacks and meals as requested. 4. Tell the client that social activities will be restricted unless food intake is increased.

2. Provide small, frequent meals. Rationale: A depressed client may eat small amounts of food because large amounts may seem overwhelming. If the client becomes overwhelmed, he or she may respond by withdrawing further. Providing snacks and meals when the client requests them will not ensure adequate nutritional intake. Forcing foods and fluids and telling the client that social activities will be restricted will cause further withdrawal by the client. Telling the client that social activities will be restricted also is a demeaning action.

The mental health nurse is reviewing the discharge plan for a hospitalized client. In reviewing the plan, the nurse recognizes that which is the most prominent problem in the management of a client with a mental health problem in the community? 1. The community's opposition 2. The client's noncompliance with medication therapy 3. The associated increased incidence of social problems 4. The family's reaction to keeping the client in the community

2. The client's noncompliance with medication therapy Rationale: Clients often forget to take their medications as scheduled, and this is the most prominent problem. Options 1, 3, and 4 may occur, but the problems described are not the most prominent and can be addressed and often controlled.

A client diagnosed with delirium becomes disoriented and confused at night. Which intervention should the nurse implement initially? 1. Move the client next to the nurse's station. 2. Use an indirect light source and turn off the television. 3. Keep the television and a soft light on during the night. 4. Play soft music during the night, and maintain a well-lit room.

2. Use an indirect light source and turn off the television. Rationale: Provision of a consistent daily routine and a low stimulating environment is important when a client is disoriented. Noise, including radio and television, may add to the confusion and disorientation. Moving the client next to the nurses' station may become necessary but is not the initial action.

The nurse is planning activities for a client diagnosed with bipolar disorder with aggressive social behavior. Which activity would be most appropriate for this client? 1. Chess 2. Writing 3. Ping Pong 4. Basketball

2. Writing Rationale: Solitary activities that require a short attention span with mild physical exertion are the most appropriate activities for a client who is exhibiting aggressive behavior. Writing (journaling), walks with staff, and finger painting are activities that minimize stimuli and provide a constructive release for tension. The remaining options have a competitive element to them and should be avoided because they can stimulate aggression and increase psychomotor activity.

A female victim of a sexual assault is being seen in the crisis center. The client states that she still feels "as though the rape just happened yesterday," even though it has been a few months since the incident. What is the most appropriate nursing response? 1. "You need to try to be realistic. The rape did not just occur." 2. "It will take some time to get over these feelings about your rape." 3. "Tell me more about the incident that causes you to feel like the rape just occurred." 4. "What do you think that you can do to alleviate some of your fears about being raped again?"

3. "Tell me more about the incident that causes you to feel like the rape just occurred." Rationale: The correct option allows the client to express her ideas and feelings more fully and portrays a nonhurried, nonjudgmental, supportive attitude on the part of the nurse. Clients need to be reassured that their feelings are normal and that they may express their concerns freely in a safe, caring environment. Option 1 immediately blocks communication. Option 2 places the client's feelings on hold. Option 4 places the problem-solving totally on the client.

The nurse is conducting an initial assessment on a client in crisis. When assessing the client's perception of the precipitating event that led to the crisis, what is the most appropriate question? 1. "With whom do you live?" 2. "Who is available to help you?" 3. "What leads you to seek help now?" 4. "What do you usually do to feel better?"

3. "What leads you to seek help now?" Rationale: The nurse's initial task when assessing a client in crisis is to assess the individual or family and the problem. The more clearly the problem can be defined, the better the chance a solution can be found. The correct option would assist in determining data related to the precipitating event that led to the crisis. Options 1 and 2 assess situational supports. Option 4 assesses personal coping skills.

The nurse observes that a client with a potential for violence is agitated, pacing up and down the hallway, and is making aggressive and belligerent gestures at other clients. Which statement would be most appropriate to make to this client? 1. "You need to stop that behavior now." 2. "You will need to be placed in seclusion." 3. "You seem restless; tell me what is happening." 4. "You will need to be restrained if you do not change your behavior."

3. "You seem restless; tell me what is happening." Rationale: The best statement is to ask the client what is causing the agitation. This will assist the client to become aware of the behavior and may assist the nurse in planning appropriate interventions for the client. Option 1 is demanding behavior that could cause increased agitation in the client. Options 2 and 4 are threats to the client and are inappropriate.

A client diagnosed with terminal cancer says to the nurse, "I'm going to die, and I wish my family would stop hoping for a cure! I get so angry when they carry on like this. After all, I'm the one who's dying." Which response by the nurse is therapeutic? 1. "Have you shared your feelings with your family?" 2. "I think we should talk more about your anger with your family." 3. "You're feeling angry that your family continues to hope for you to be cured?" 4. "You are probably very depressed, which is understandable with such a diagnosis."

3. "You're feeling angry that your family continues to hope for you to be cured?" Rationale: Restating is a therapeutic communication technique in which the nurse repeats what the client says to show understanding and to review what was said. While it is appropriate for the nurse to attempt to assess the client's ability to discuss feelings openly with family members, it does not help the client discuss the feelings causing the anger. The nurse's attempt to focus on the central issue of anger is premature. The nurse would never make a judgment regarding the reason for the client's feeling; this is nontherapeutic in the one-to-one relationship.

When the mental health nurse visits a client at home, the client states, "I haven't slept at all the last couple of nights." Which response by the nurse illustrates a therapeutic communication response to this client? 1. "I see." 2. "Really?" 3. "You're having difficulty sleeping?" 4. "Sometimes, I have trouble sleeping too."

3. "You're having difficulty sleeping?" Rationale: The correct option uses the therapeutic communication technique of restatement. Although restatement is a technique that has a prompting component to it, it repeats the client's major theme, which assists the nurse to obtain a more specific perception of the problem from the client. The remaining options are not therapeutic responses since none encourage the client to expand on the problem. Offering personal experiences moves the focus away from the client and onto the nurse.

A client is admitted to the mental health unit after an attempted suicide by hanging. The nurse can best ensure client safety by which action? 1. Requesting that a peer remain with the client at all times 2. Removing the client's clothing and placing the client in a hospital gown 3. Assigning a staff member to the client who will remain with the client at all times 4. Admitting the client to a seclusion room where all potentially dangerous articles are removed

3. Assigning a staff member to the client who will remain with the client at all times Rationale: Hanging is a serious suicide attempt. The plan of care must reflect action that ensures the client's safety. Constant observation status (one-to-one) with a staff member is the best choice. Placing the client in a hospital gown and requesting that a peer remain with the client would not ensure a safe environment. Seclusion should not be the initial intervention, and the least restrictive measure should be used.

A client is admitted to a medical nursing unit with a diagnosis of acute blindness after being involved in a hit-and-run accident. When diagnostic testing cannot identify any organic reason why this client cannot see, a mental health consult is prescribed. Which condition will be the focus of this consult? 1. Psychosis 2. Repression 3. Conversion disorder 4. Dissociative disorder

3. Conversion disorder Rationale: A conversion disorder is the alteration or loss of a physical function that cannot be explained by any known pathophysiological mechanism. A conversion disorder is thought to be an expression of a psychological need or conflict. In this situation, the client witnessed an accident that was so psychologically painful that the client became blind. Psychosis is a state in which a person's mental capacity to recognize reality, communicate, and relate to others is impaired, interfering with the person's ability to deal with life's demands. Repression is a coping mechanism in which unacceptable feelings are kept out of awareness. A dissociative disorder is a disturbance or alteration in the normally integrative functions of identity, memory, or consciousness.

A depressed client verbalizes feelings of low self-esteem and self-worth typified by statements such as "I'm such a failure. I can't do anything right." How should the nurse plan on responding to the client's statement? 1. Reassure the client that things will get better. 2. Tell the client that this is not true and that we all have a purpose in life. 3. Identify recent behaviors or accomplishments that demonstrate the client's skills. 4. Remain with the client and sit in silence; this will encourage the client to verbalize feelings.

3. Identify recent behaviors or accomplishments that demonstrate the client's skills. Rationale: Feelings of low self-esteem and worthlessness are common symptoms of a depressed client. An effective plan of care to enhance the client's personal self-esteem is to provide experiences for the client that are challenging, but that will not be met with failure. Reminders of the client's past accomplishments or personal successes are ways to interrupt the client's negative self-talk and distorted cognitive view of self. Options 1 and 2 give advice and devalue the client's feelings. Silence may be interpreted as agreement.

The nurse is developing a plan of care for a client who was experiencing anxiety after the loss of a job. The client is now verbalizing concerns regarding the ability to meet role expectations and financial obligations. What is the priority problem for this client? 1. Anxiety 2. Unrealistic outlook 3. Lack of ability to cope effectively 4. Disturbances in thoughts and ideas

3. Lack of ability to cope effectively Rationale: Lack of ability to cope effectively may be evidenced by a client's inability to meet basic needs, inability to meet role expectations, alteration in social participation, use of inappropriate defense mechanisms, or impairment of usual patterns of communication. Anxiety is a broad description and can occur as a result of many triggers and although the client was experiencing anxiety, the client's concern now is the ability to meet role expectations and financial obligations. There is no information in the question that indicates an unrealistic outlook or disturbances in thoughts and ideas.

The emergency department nurse is caring for a client who has been identified as a victim of physical abuse. In planning care for the client, which is the priority nursing action? 1. Adhering to the mandatory abuse-reporting laws 2. Notifying the case worker of the family situation 3. Removing the client from any immediate danger 4. Obtaining treatment for the abusing family member

3. Removing the client from any immediate danger Rationale: Whenever an abused client remains in the abusive environment, priority must be placed on ascertaining whether the client is in any immediate danger. If so, emergency action must be taken to remove the client from the abusing situation. Options 1, 2, and 4 may be appropriate interventions, but are not the priority.

A client is being prepared for electroconvulsive therapy (ECT). The nurse's plan of care for the day before ECT includes ensuring that the client follows which guideline? 1. Does not smoke at all 2. Receives no visitors and participates in limited unit activities 3. Reports to the clinic for blood draws and an electrocardiogram (ECG) 4. Is placed on nothing by mouth (NPO) status for 16 to 24 hours before the ECT

3. Reports to the clinic for blood draws and an electrocardiogram (ECG) Rationale: Before ECT, blood tests are performed and an ECG is done to determine a baseline status of the client. The nurse needs to explain the need for these preprocedures to the client. Maintaining *NPO status for 6 to 8 hours before treatment* is adequate; NPO status for 16 to 24 hours is not necessary. Some hospitals place clients on NPO status at midnight before ECT in the morning. Some clients who are on cardiovascular medication may be instructed to take their medicine with sips of water several hours before ECT. Options 1 and 2 are incorrect.

The nurse is monitoring a client with anorexia nervosa. Which statement by the client would indicate to the nurse that treatment has been effective?

"My friends and I went out to lunch today."

The most important tool of psychiatric nursing is the:

Self

medication for anxiety in children

duloxetine


Ensembles d'études connexes

Summarizing Literature: Mastery Test

View Set

Ch. 5 analyzing the marking enviorment

View Set

Microeconomics Chapter 1: Limits, Alternatives, and Choices

View Set

MH personality and mood practice ch 18

View Set

Medsurg dynamic adaptive quizing

View Set

Microeconomics Smartwork_5 Assignment

View Set